You are on page 1of 51

1

2012 SRJC JC2 H2 Economics Mid-Year Examination Paper 2


Answers, Marking Scheme & Examiners Comments for Students

The cruise industry is a highly profitable international activity and the fastest growing
sector of the travel, tourism and leisure industry. Cruise prices remain in consumers
favour going into 2012.
(a)

Explain the economic principle that underpins the pricing policy of a firm in the
perfect and imperfect market structures.
[10]

(b)

Discuss whether it is a valid economic argument that marketing is more


important than pricing policies for a cruise operator.
[15]

Suggested answer outline: Qn1 (a)


INTRODUCTION
(i) Define perfect and imperfect market structures: A perfect market structure is
characterised by many small firms selling an insignificant proportion of a
homogeneous product such as an agricultural produce like corn in the industry. An
imperfect market structure on the other hand is characterised by firms with some
degree of market power arising from selling differentiated products.
(ii) State direction of answer: The traditional theory states that all firms seek to
maximise profits. The economic principle that underpins a firms pricing policy is to
price his product at the profit maximising level of output, given its cost and market
conditions. This occurs if his marginal cost of production equals to his marginal
revenue gained from selling the last unit for the price that he charges for his output.
This same principle guides pricing in both market structures as explained below.

P1

BODY
(1) State that the profit maximising rule of MC=MR and where MC is rising must be
adhered to.
The profit maximising rule of MC=MR is the economic principle that forms the
basis of a firms pricing. This determines the profit maximising output and the
price that all firms whether perfect or imperfect should charge in order to maximise
total profits. This occurs when the cost of producing the last unit is equal to the
revenue earned from selling it, that is MC=MR. This is known as the P=MC pricing
rule in the perfect market
(I) Briefly explain the construction of a PC firms marginal Revenue (MR) and
marginal cost (MC)curves.
(i) Establish the MR curve of a PC firm as a horizontal straight line.
In the perfect market structure, price OP is determined by the industry demand
for and supply of the product in figure 1a. The perfectly competitive firm is a
price taker with no market power to change the industrys price as it is only one
of many firms producing only a small proportion of the industry output. Because
the price is fixed at OP, e.g. $5, it is also equal to the average ($5) and marginal
revenue ($5) earned from selling one more unit. Hence the firms price OP
equals to his average revenue (AR) and MR curve in figure 1b.

SRJC

Price

Price, AR, MR
MC
S

AC
MC=MR

AR/MR
a

D
Q

Quantity

Qo

Fig 1a: PC Industry

Qe
Q1
Fig 1b: PC firm

Quantity

(ii) With given price, that is MR, the PC ensures that it adjusts its output till its
MC is equal to MR/price. Hence this profit maximising principle illustrates
a price equals to marginal cost pricing policy in the case of a perfectly
competitive firm
(a) This is because for output levels less than 0Qe, MR/P > MC, there is potential for
more profits to be earned. This is due to the fact that the addition to total revenue
earned from the sale of the last unit of output is greater than the additional cost
incurred in producing it. Thus, the firm can in fact increase its total profits if it
increases the level of production.
(b) For output levels beyond 0Qe, the sale of the last unit of output adds less to total
revenue than the additional cost incurred in producing it. Thus, producing output
beyond 0Qe results in a fall in total profits. Hence, profit is only maximised when the
last unit of output produced adds as much to total revenue as it does to total cost,
i.e. where MR=MC=P.
It is pertinent to note that at 0Q0, marginal revenue is equal to marginal cost, but
marginal cost is falling. Thus, if the firm continues to expand its output beyond
0Q0, its profits will increase because marginal revenue exceeds the marginal cost for
this range of output. At 0Qe, mc=mr and mc is rising.
L

P2

Hence we can see that a profit maximising PC firm will equate its price with its MC at
equilibrium. In addition, this principle hinges on the fact that his AVC is at least
covered in short run. In fig 1(b), it makes a profit of $Ea per unit. In the long run, its
price must at least cover all average costs of production.
(II)

Similarly, a firm in the imperfect market will also set a profit maximising
price, which is underpinned by the MC=MR rule. However, unlike PC firm,
the profit maximising price 0P is higher than the marginal cost of producing
the last unit, i.e. P>MC.
Price, Revenue, Cost
MC
P

AC

MR
0
SRJC

Q1

DD = AR
Quantity

3
In an imperfect market structure where monopolistic firms sell differentiated products
or where a monopolist operates as sole seller, some degree of market power to
either control output or price exists. To maximise profits, he will also charge a price
where MC=MR and MC is rising. In figure 2, it can be seen that his MR is lower than
his AR for every additional unit sold. This is because in order to sell more, it can
reduce its price. Hence his MR is not equal to a fixed price, but lower than the price
for every additional unit sold. Hence the profit-maximising price OP in Figure where
profit maximising output OQ occurs will be such that P>MC, since MR is less the
price.
If demand for his product changes and his MR rises, he will adjust output accordingly
to equate MC equal to his new MR and price accordingly based on the new AR at
the new eqm output.
The diagram shows an imperfect firm making supernormal profits of area PGFH with
a profit maximising price at 0P.
In the long run, its P>AC because barriers to entry prevents supernormal profits from
being competed away in the case of a monopoly.
(III)
Other considerations:
Firms may sometimes pursue the goals of revenue maximisation and price
goods to earn the highest possible sales/total revenue rather than total
profits.
CONCLUSION
Nevertheless, the key pricing policy of a firm stems from the basic profit maximising
output policy. This determines price such that P=MC=MR in a perfect market structure
but a P >MC because profit maximising MC=MR where MR<P. In the short run,
P=MC=AVC and P=MC=AC.
L3

L2

L1

Rigorous explanation of profit maximizing condition of MC=MR and


development of the PC/imperfect model in analysis.
Considered both MC>MR and MC<MR and charging a P=MC and P>MC
in different market structure.
Diagrams are detailed and well explained. Falling portion vs rising portion
of MC in eqm and the AC curves are highlighted as well.
Explained sufficiently MC=MR principle in max profits and corresponding
price
Considered either MC>MR and/or MC<MR well
Diagram is attempted and explained with reference to the PC model or
imperfect market model.
Some observations concerning whether P=MC pricing or P is >MC as
profit maximising price.
Splattering of relevant and irrelevant points.
Definitions of market structure, PC firm, basic concepts and principles.
Mere statements of profit maximising conditions.

(b)
Discuss whether it is a valid economic argument that marketing is more
important than pricing policies for a cruise operator.
[15]
INTRODUCTION
State assumptions about aim of cruise operator: The aim of a cruise operator is to earn
highest possible total profits.
State that in order to raise total profits, a firm must reduce total cost of producing the
service and increase TR from sales of its service. Conversely, total revenue must
increase by a greater extent than the increase in total cost.

SRJC

4
State that the cruise operator can increase total revenue by changing the price of its
services or influencing non-price demand factors to increase the demand for its service
at the same price.
State direction of answer/yardstick for evaluation: Whether pricing strategy or marketing
is more important depends on cost of such strategies, the market structure in which it
operates and the time period under consideration.

P1

BODY
Thesis:
The cruise industry is likely to be oligopolistic dominated by a few large firms
because of high barriers to entry. This is because the business requires high
capital investments such as a cruise liner. For such a market structure, nonpricing strategies such as marketing and promotion is more effective than price
strategies may to increase profits.
This is because a cruise operator may want to avoid price competition and a price war
as explained by the kinked demand theory. This theory arises from mutual
interdependence among the few firms, for example in Singapore firms such as Star
Cruise and Superstar Virgo. This means that firms watch out for the actions of its rivals
and react accordingly. Figure 1 illustrates the kinked demand AEF where the portion AE
is price elastic and EF is price inelastic. The kink occurs at price 0P. For example, if an
operator, eg Star cruise reduces its price from 0P to increase its market share,
competitors may follow suit to prevent a loss of its market share.
Figure 1

Price, revenue,
cost
A
P

LRMC

LRAC
A
0

F
Q

MR

Output

Hence, a fall in price for Star Cruise will only increase quantity demanded for its
services by a smaller extent and cause a fall in its total revenue. This is because the
gain in total revenue from the smaller percentage rise in quantity demanded is unable to
offset the loss in revenue from the greater fall in price. Similarly if it raises its price,
competitors are assumed not to follow suit in order to hold on to its market share.
Hence, if Star increases its price, a greater percentage fall in quantity demanded will
result. Hence the loss in revenue from the greater percentage drop in volume of sales
cannot offset the gain in revenue from the rise in its price/AR.
Hence, assuming total cost unchanged, such pricing changes where the kink occurs will
only cause total profits to fall.
Unless the cruise operator colludes with the few rivals firms in the industry and set an
industry price to ensure profit maximising, the cruise operator should focus on creating
strong demand for its product based on differentiating its services.

P2

Hence in order to compete successfully, a cruise operator can affect demand


positively by increasing their willingness to go on their cruise holidays through
marketing strategies such as through promotion and advertising programmes

SRJC

5
that aim to differentiate their services.
E

For example, to attract more customers, the operator can advertise aggressively and
sell new travel experience such as sports, theatrical performances, exotic food cuisines
on cruises. It can also plan for family cruises. Its marketing strategies can also aim to
reach more customers through eg fairs, internet booking, etc.
Hence if marketing is successful, the demand for its cruises at the same price will rise.
Figure 2 shows the original demand curve D for cruises provided by a firm, e.g. firm A.
Successful marketing results in a rightward shift from D to D1.

P3

At the same time, if marketing brings about brand loyalty, the demand for its
services will also become more price inelastic. Hence the new demand curve D1
also becomes steeper in the same diagram
Cost, Revenue

MC
C

P1
A

P
D

AC

F
B
AR1= D1

E
MR1
Figure 2

MR
Q Q1

AR=D
Output

The profit maximising output therefore increases to 0Q1 where MC=MR1. Assuming
average cost curve remains unchanged, we can see that the average profits increases
from $AB per unit to $CF per unit. The cruise operator should ensure that if promotion
costs are high and average cost rises as a result, the marketing programme must be
such that demand must rise by a great extent to ensure that total revenue earned from
sales of cruises will be greater than total cost of providing them.
L

Hence we can see that marketing programmes to promote the business in place
of pricing strategies must be undertaken. Increasing demand and making its
demand more price inelastic through marketing and promotion is deemed
necessary to sustain its growth in the long run.

P4

Anti-Thesis: However, although the theory of oligopolistic behaviour suggests an


avoidance of pricing strategy, as long as the firm is able to reduce price due to e.g. new
and cheaper technology in operating the cruise business, the firm in reality should do so
to increase sales revenue. In addition, if marketing makes his services more
differentiated as in above, he can consider raising his price to increase sales revenue
and profits. If oligopolistic firms operate where his MC rises beyond the MR discontinuity
prices may need to be changed accordingly. In some cases of collusion, pricing
strategies in the form of price leadership can occur. Similarly, if the cruise industry is
characterised by monopolistic competition, pricing strategies must also be used.

For example pricing strategies may be discussed in terms of lowering the price for

SRJC

6
cruise travel. The demand for cruise travel may be assumed to be price elastic,
because holiday trips are deemed to be luxury items and cruise holidays have other
substitutes such as within the cruise industry and across the travel industry such as
having a holiday programme largely based on air travel from destination to destination.
Hence ceteris paribus, if a cruise operator lower the price of its services, his total
revenue will rise.
Price

P
C
P1

D
B

A
Figure 3

quantity of cruise travel


0

Q1

Figure 3 shows the downward sloping demand curve for a normal good like cruise
travel. This shows that when price of a cruise falls, ceteris paribus, cruise travel
becomes cheaper which increases the willingness and ability of holiday makers to go for
more cruise holidays.
At original price 0P, the quantity demanded is 0Q in figure 1. Hence the total earnings is
0PEQ, which is derived by multiplying price by the quantity demanded.
It may be assumed that the demand for the operators cruise travel is price elastic due
to existence of a number of cruise operators which offer similar cruises within same
price range, such as STAR cruises and Caribbean. Hence when a firm A, e.g. STAR
company reduce its price, ceteris paribus (e.g. taste and preference and income levels
of travellers remain unchanged), there will be a greater percentage rise in quantity
demanded to 0Q1.

P5

Hence the gain in total revenue of area B will be greater than the loss of area C as a
result of the smaller percentage drop in price. The net effect is a rise in total earnings for
the cruise operator. Ceteris paribus, i.e. cost remaining unchanged, the cruise
operators total profit will rise with such a pricing strategy.
Hence we can see that marketing programmes to promote the business in place of
pricing strategies is deemed necessary to sustain its growth of in the long run. In
addition, as long as although the theory of oligopolistic behaviour suggests an
avoidance of pricing strategy, as long as the firm is able to reduce price due to e.g. new
and cheaper technology in operating the cruise business, the firm in reality should do so
to increase sales revenue.
CONCLUSION
Hence, although it is valid argument that marketing strategies are important, it may not
be necessarily be more important than pricing strategies. Cruise operators should not
depend on only one strategy but use both marketing and pricing strategies effectively to
raise the profits of its cruise business.

SRJC

7
Level

Descriptors

L4

(i) Explained and discussed thoroughly both price elasticity of demand and
marketing strategies on the total revenue and profits of a cruise operator
(ii) Competent use of well labelled diagrams
(iii) Good use of examples
(iv) Provided insights on the cruise industry with good use of economic models on
types of firms in explanation and evaluation, e.g. whether operating in perfect
or imperfect market structures.
(v) Cohesive discussion on the relative importance of the 2 strategies

L3

(i) Applied factors affecting price elasticity of demand and explained clearly price
elasticity of demand on cruise business. Some evaluation of price elasticity on
cruise business.
(ii) Explained and evaluated marketing strategies on total revenue and profits for
a cruise operator sufficiently
(iii) Some comparison of the 2 measures
(iv) Diagrams are labelled, explained and used to illustrate explanation well.

L2

(i) Some explanation of price elasticity and non-price demand factors affecting
cruise travel.
(ii) Some explanation of effects of both concepts on total revenue and profits
earned by a cruise operator.
(iii) Diagrams are used to illustrate explanation.
Smattering of points on demand curve, demand factors and objectives of a firm.

L1

SRJC

8
Examiners comments:
1
The cruise industry is a highly profitable international activity and the fastest growing sector of the
travel, tourism and leisure industry. Cruise prices remain in consumers favour going into 2012.
(a)

Explain the economic principle that underpins the pricing policy of a firm in the perfect and
imperfect market structures.
[10]

(I)Question Interpretation/Requirement
1. This question tests a fundamental understanding of the economic principle of the profit maximising
rule for firms where their pricing is concerned.
Essentially, it asks how firms determine what price to charge to bring in the highest possible profits in
terms of. The conditions that must be met to ensure that it will sell the output that profit maximises.
2. Firstly, the answer requires a recognition and a clear statement that a profit maximising firm will set a
price based on a profit maximising output which occurs where the firms mc= mr. In the short run, his
average revenue or price must cover average variable costs and in the long run, it must cover all costs.
Hence the fundamental economic principle that underpins/supports/forms the basis of pricing is
mc=mr
(i) For the pc firm, it is called mc pricing where he equates the given price to his mc of the last unit
produced.
(ii) In contrast, this profit maximising price for the firm operating in the imperfect market occurs where
his P>MC when profit is maximised.
3. Essentially, it requires you to then explain this (above) principle to convince why it is so. Note that
stating a principle is not an explanation. This question essentially asks you to unpack /explain this
principle clearly.
Common Errors
1 Many candidates did not understand or appreciate the fundamental principle and wrote that characteristics is
the economic principle that determines pricing.
A principle is a rule or law. Whilst characteristics can influence pricing as to whether price is charged at mc or
above mc and whether it is a price taker or setter, it itself is not an economic principle. (The Question is not
phrased as what are the factors that can influence pricing or market power).
Do note the difference between the word principle and principal The former is about a rule, the latter is
another word for main or most important eg: discuss whether labour productivity is the principal factor for
ensuring sustainable economic growth in a country like Singapore. This requires a discussion on other factors as
well. besides labour productivity
2. Some candidates used the concept of PED elasticity to explain whether a firm should increase or reduce price.
Whilst price may affect his pricing options, the key principle or rule that forms the basis of this pricing is still
profit maximising, that is, if he raises his price based on PED<1. it still about earning the highest profits with a
rise in TR. If the candidate can recognise this, it can be credited accordingly.
3. Some candidates wrote randomly on maximising society welfare where P=MC.
This is irrelevant to the question requirement. The question asks about a firms basic aim, not societys
(II)Clarity and Precision: Content Analysis and Application
What you wrote
Hazy concept of losses and profits

1.

If the firm produce beyond mc=mr, it will make a


loss

2.

Imprecise features of PC firm


The firm in the perfect market sells products
that are close substitutes with their competitors

SRJC

What it should be
This is not necessary true or accurate.
What you should write is that if it produces an output
beyond mc=mr where mc is rising, profits will be
less than at mc=mr where mc rising.
In order words, it can make profits, but it is not the
highest possible.
The total average level of profits or loss will depend
on the AR and AC (which includes all costs)
conditions. Alternatively, calculate the TR and TC to
arrive at total profits at this profit maximising point.
Wrong conceptually.
The correct term you must use is perfect or
identical substitutes.
Close means similar and differentiated

9
3.

Some confusion in the technical construct of


a PC firm and PC industry

This was disappointing


Note the basics that: The industry is made up of
firms producing the same product. So as a basic rule,
the demand for the industrys product as a whole or
as a market itself is downward sloping. Price must
fall for the total consumption to increase in the
market.
This price is determined in the market. The
individual firm takes the price as determined in the
industry. Hence the price is fixed and is represented
by a horizontal straight line.

(III)Quality of answer
There is big room for improvement in terms of scope and depth.
There is relevant amongst irrelevant writing.
(IV)Organisation of answer
There is attempt to organise and structure paragraphs with a key topic sentence although the key topic sentence
was often not effective or spot on. The answer needed to be sussed out by the reader.
(b)

Discuss whether it is a valid economic argument that marketing is more important than pricing policies
for a cruise operator.
[15]

(I)Question Interpretation/Requirements
This question is largely about shifts and movements of the demand curve and impact on total revenue and total
cost for the cruise operator. The quality of treatment of the question requirements amongst differed
The question required
(i)a framework and introduction in terms of a firms basic objective to earn the highest possible profits
(ii)a recognition that marketing is a non-price determinant that shifts the demand curve at the same price and
pricing is a price determinant involving a movement along the same demand curve and a discussion price
elasticity on total revenue.
(iii) An analysis of the impact on total revenue and total cost of the 2 approaches was required.
(iv) In addition, an evaluation and comparison of the two approaches with a conclusion.
Good answers made good assumptions about the market structure in which they placed the cruise industry
(II)Clarity and Precision/Content Analysis and Application

1.

What you wrote


Unexplained conclusions.

Marketing and promotion can make demand more


price inelastic, hence more important
2.
Erroneous and unnecessary relationship in
stating a precise proportionate relationship
between PED and TR.
If the demand is price elastic, the total revenue will
rise proportionately when there is a price fall.

3.

Limited explanation of the kink demand curve

SRJC

What it should be
The link between PED<1 and TR must be explained.

Yes, total revenue will rise since the percentage


change in the quantity demanded is more than the
percentage fall in price, ceteris paribus.
However, whether total revenue rises proportionately
or not depends. Firstly, it is not clear what the
candidates are comparing the proportionate change to.
If it is to compare the extent of the change in total
revenue itself compared to price change, the only
mathematical possibility for total revenue to rise
proportionately, that is proportionate to the price
change is when demand is perfectly price inelastic.
Hence it is not true that if demand if price elastic, total
revenue will rise proportionately. Avoid such
complexity that is not required in the answer. Just
concluded that total revenue will rise.
You need to explain clearly the impact on a drop in
TR with price changes and the final impact on profits

10
The PED is high above the price at the kink,
and <1 below the kink

4.

Irrelevant and deviation from question


requirements
Marketing leads to wastage of resources and
inefficiency in resource allocation from society's
point of view

(i)

The kink demand curve was largely


badly drawn and incomplete.
(ii)
you should also superimpose the cost
curve for discussion.
Only 5 candidates managed to draw an imperfect
market structure cost and revenue curve when
discussing how marketing shifts the AR and makes it
more price inelastic
What you could write is on the cost considerations.
and effectiveness of marketing on the firm. This
depends on whether cost of advertising itself is more
than compensated for by rise in TR as a result of the
shift in demand

(III) Quality of answer


There could have been more competent use of a correct model of analysis either for imperfect market or at least
a comparison between movement and shift and a net effect of both.
(IV) Organisation/Structure of answer
There was sufficient evidence of a thesis, anti-thesis and synthesis approach in answering the question. In other
words, there was analysis and evaluation and a conclusion.

SRJC

11
Examine the relative impact of a deep recession and a government subsidy on total
consumer expenditure in different markets such as food and the Arts and Cultural
events.
[25]

INTRODUCTION:
(1) State that:the market is determined by demand for and supply of the product.
(2) State that total consumer expenditure is derived by multiplying price per unit
and total quantity bought by the consumer.
(3) Recognise that a deep recession and subsidy can affect the demand for and
supply of the product and hence equilibrium price and quantity bought.
(4) State direction of answer: the extent of change on the 2 different markets and
hence total expenditure depends on the nature of the product and price and
income elasticity of demand.
BODY
(1) Explain the impact of a deep recession on the market for food as a broad
category
P1

A deep recession will affect the market and cause a fall in the total consumer
expenditure for food items.

Figure 1 below shows the downward sloping demand (D) and upward sloping
supply (S) curve for a normal good such as food items, for example rice. Ceteris
paribus, the lower the price, the higher the quantity demanded by the consumer.
Conversely, the higher the price, the greater the incentive for the profit maximising
producer to increase quantity supplied. The original market equilibrium is
determined where demand equals to supply at point E. The equilibrium price and
quantity exchanged is at 0P and 0Q respectively. The total consumer expenditure is
represented by area $0PEQ.
price per
unit ($)

P
P1

E1

D
D1

Figure 1

quantity of food
0

Q1

A deep recession would mean that there will be a great fall in the income earned by
the citizens of the country. This reduces the disposable income and spending
power of the citizens. Ceteris paribus, for example, price of the good remaining
unchanged, there will be a fall in the willingness and ability of households to
demand for a normal good.

As food as a broad classification are necessities needed for basic survival Hence,
other factors remaining unchanged, when there is a fall in income, households will
reduce the quantity consumed only by a small extent. This means that food has a
low positive income elasticity of demand. This means, for example, for a given
5% fall in income, the demand will fall by less than 5%. Hence, the demand curve

SRJC

12
will shift to the left by a smaller extent to D1.
As a result, a surplus arises which exerts a downward pressure on equilibrium
price. As price falls, the quantity demanded increases causing a movement along
D1. At the same time, producers seeing the drop in price will reduce quantity
supplied until a new equilibrium is re-established at E1. With a lower equilibrium
price at 0P1 and lower quantity bought at 0Q1, total expenditure of consumers falls
to $0P1E1Q1.
P2

However, as at the same time, the government grants a subsidy e.g. on the
production of rice and other basic food items. Hence, there will be a further
influence on food consumption during the recession.

A subsidy on rice production, for example, reduces cost of production and enables
rice farmers to produce more rice. Hence in figure 2 below, at the same price of
rice, there will be a simultaneous rightward shift of the supply curve from S to S1.
Alternatively, it can be seen that supply curve shifts vertically downwards by the
amount of the subsidy per unit. Hence, food prices will become cheaper as the
lower cost of production may mean that the producer will pass over this cost saving
to consumers in the form of a lower price. Ceteris paribus, the fall in price increases
quantity demanded along D1.
The change in total expenditure will depend on the degree of responsiveness of
quantity demanded given this price fall. The demand for a necessity like food items
is not very sensitive to price changes, so a cheaper price will only bring about a
smaller percentage rise in quantity demanded. Hence, in figure 2, the rise in total
expenditure denoted by area B will be smaller than the fall in total expenditure of
area C due to the smaller percentage fall in price. Hence total expenditure (A+B) is
smaller than A+C. Hence total expenditure by consumers on food shrinks.
Price per unit ($)
S

S1 (S+ subsidy)

P
P1
C

Es

Ps
A

B
D1

Fig 2
0

Q1

quantity of food items

In addition, as agricultural produce requires a longer gestation period, the supply is


price inelastic. Hence coupled with a price inelastic demand, a greater percentage
drop in price results when there is a surplus. With a smaller percentage rise in final
equilibrium quantity exchanged due to a demand that is price inelastic, the total
expenditure is likely to fall by a great extent.

Hence the 2 effects reinforce one another and the net effect of a deep recession in
spite of a subsidized price is a significant fall in total expenditure on food. The total
expenditure $0PEQ has fallen to 0PsEsQ.

SRJC

13
P3

On the other hand, services like Arts and Cultural events are normal goods which
are likely to be luxury items. These are leisure activities and could include pop
concerts, museum and other theatre performances either by local or foreign acts.

Hence, like food which is a normal item, during recession where income falls, there
will also be a fall in the demand for Arts and Cultural events. However, as these are
luxury goods which households can forgo during hard times and will spend on after
basic needs are met, there will a greater percentage drop in demand compared to
the fall in income. Hence, the demand curve for Arts and Cultural events shifts to
the left by a greater extent. Assume supply remaining unchanged, the surplus
causes the same automatic market adjustment process through a downward
pressure on price as explained earlier in the food market. Ceteris paribus, the
bigger surplus will cause a greater fall in equilibrium price and quantity exchanged
for such events. Hence with a smaller price and quantity bought, the impact is a
greater fall in total expenditure to area 0PAEAQA compared to food expenditure of
area 0PfEfQf as seen in the figure below.
price per unit
($)

P
Pf

PA

EA

P4

Ef

QA Q f

Df
Df1
Q

Qty of Art Events


& food

If a subsidy is provided by the government to support such an industry and to


stimulate its long-term growth potential, this will encourage more firms to join the
industry and promote such consumption. This enables such events to be affordable
for more people. Whether it can increase consumption significantly depends on the
price elasticity of demand for such goods. In such a market, the degree of
responsiveness of demand will depend on the particular type of arts and cultural
events and whether there are good substitutes for leisure activities. For example,
other factors remaining constant, with a subsidised price for Arts and Cultural
events, households will be more willing and able to pay to attend concerts and likely
to switch from staying at home for other alternative entertainment such as watching
television
By and large, the demand for a luxury good would be more price elastic than that
for food and would increase total expenditure as a result of a greater percentage
increase in quantity demanded for a given price fall arising from the subsidy.
As seen in the diagram below, the demand curve shifts from D to D1 while the
supply curve shifts from S to S1. In this case, the subsidy causes a rise in consumer
expenditure when price fell from 0P to 0P1.

SRJC

14

price
S
P

S1 (S+ subsidy)

E
E1

P1

D
D1

Fig 4

Q1

quantity of arts and


cultural events

Similarly the responsiveness may depend on age groups. If citizens do not place
much value on such cultural events, then the percentage rise in quantity demanded
may be insignificant. Hence the expenditure will fall. Coupled with the fall in
consumption due to a fall in income, the net effect is also a fall in total consumer
expenditure. If the demand is of unitary price elasticity, the total expenditure
remains unchanged with a subsidized price. The net effect is therefore a fall in total
expenditure.
The combined impact of the deep recession and subsidy may be either a rise or a
fall in total expenditure depending on whether the rise in expenditure due to a
subsidised price is able to offset the fall in expenditure due to a fall in his income.
Comparison of the impact on food and arts and cultural events
Hence we can see that the impact on the expenditure on both goods work through
a shift in demand and a movement along the demand curve. There are 3 possible
final impact of the 2 triggers: Both will see a fall in total expenditure. Both work to
reduce consumption expenditure where a deep recession is concerned. Assuming
PED<1 for both, then both fall. If PED=1 for Arts, there will be a net fall in total
expenditure.
In view of the fact that food items are necessity with low income and price elasticity
of demand, the final impact of both a deep recession and a subsidy may cause a
bigger fall in total expenditure in food items than for the arts and cultural events.
This is because both the recession and subsidy bring about a fall in total
expenditure for food.
(optional analysis) However, if the food item under examination is an inferior good
such as white rice as compared to brown rice, the impact of a recession could be to
increase total consumer expenditure. This is because inferior goods have a
negative income elasticity of demand. Hence during a deep recession, households
switch from better quality rice to poorer substitutes. Figure 5 shows a rise in
demand to D1 where the quantity bought has increased to 0Q1. Hence the
consumer expenditure for such food items will increase, ceteris paribus.
At the same time, if the government intervenes by granting a subsidy, it could still
bring about a rise in total consumption expenditure if the rise in total expenditure
due to a rise in demand is not offset by the fall in total expenditure as a result of a
subsidy. The new equilibrium price and quantity is 0P1 and 0Q1 respectively.
Hence, total consumer expenditure rises from 0PEQ to 0P1E1Q1.

SRJC

15

price
S

S1 (S+ subsidy)
E

P
P1

E1
D1
D

Fig 5
0

Q1

quantity of inferior food


items

However, there are the limitations this examination as it hinges on the ceteris
paribus assumption. Other factors like changes in life style and taste and
preference could have changed at the same time. These can affect the degree of
change in the willingness and ability of households to consume a good when
income and price changes.
CONCLUSION
Hence, a closer examination suggests that the relative impact depends on the
nature of the good. A strong consideration is the fact that the recession is deep and
is likely to cause a fall in total expenditure for both goods. Hence if government
wishes to increase total expenditure to boost an economy, it may have to find other
ways of doing so, for example by reducing taxes and increase their disposable
income.

Level
5






4







SRJC

Descriptors
Excellent understanding and analysis of market equilibrium and factors
affecting price and income elasticity of demand of the 2 goods.
Lucid and rigorous analysis of the effects of a deep recession and subsidy.
Lucid evaluation of the net effects on total consumer expenditure of both
trigger on the respective good.
Good scope in terms of analysis on different types of goods within the
category.
Diagrams are well labelled and competently used to illustrate answer.
Cohesive answer with competent comparison of the relative impact on
consumer expenditure in the 2 markets.
Good understanding and analysis of market equilibrium and factors affecting
price and income elasticity of demand of the 2 goods.
Rigorous analysis of the effects of a deep recession and subsidy.
Good evaluation of the net effects on total consumer expenditure of both
triggers on the respective good.
Some scope in terms of analysis on different types of goods within the
category.
Diagrams are well labelled and competently used to illustrate answer.
Some recognition of the difference in impact based on nature of the
market/goods.
Good understanding of market equilibrium and price adjustment process as
triggered by the 2 factors given.

16
Good explanation and application of price and income elasticity of demand in
one market and effects on total consumer expenditure; explanation and
application to the other market is not consistently good
 Some examination of the relative impact of the net effect on both goods.
 Good consideration of examples.
 Good use of well labelled diagrams.
Some understanding of concept of market equilibrium, total consumer expenditure
and effects of the 2 triggers on the final price and quantity exchanged.
 Some explanation of effects of recession on the shift of demand in the 2
different markets and expenditure by consumers.
 Some explanation of effects of a subsidy on shift of supply curve and effects
on price and expenditure.
 Some examination of net effects on total consumer expenditure of the 2
triggers.
 Diagrams are drawn and sufficient reference made in answer.
 Smattering of points on markets, equilibrium, shifts and movements and
expenditure on the different types of goods.
 Conceptual error/s evident.


SRJC

17
Examiners comments:
2
Examine the relative impact of a deep recession and a government subsidy on total consumer
expenditure in different markets such as food and the Arts and Cultural events.
[25]
(I)Question Interpretation / Requirement
1.
Examine relative impact means to compare the impacts.
Note that Question asks for comparison of the impact in terms of total expenditure, not total quantity
purchased (qty exchanged)
2.
The examination essentially requires an application of both income and price elasticity of demand and
the nature of the good. A net effect of both triggers on food and Art and Cultural events respectively
and comparison of the net effects between the 2 good is necessary.
Strength:
There were some insightful evaluations based on clear technical analysis.
Common errors.
1. end point was lost in the analysis for weaker attempts
2. This is partly due to
(i)
not clarify the term total expenditure in the introduction.
(ii)
misconception of the term to mean quantity or volume bought
(iii)
reading of the question as a question on impact on the market which in itself would mean
changes in the equilibrium price and quantity exchanged.
(II) Clarity and Precision/ Content Analysis and Application

1.

2.

3.

What you wrote


Casual, vague and inaccurate writing
During a recession, people will still buy food
as it is a necessity. People still have to eat.

Concept errors and inaccuracy


The demand for food will fall because it is a
necessity whilst that for Art and Cultural
events will rise because it is a normal good

Lack of explanation
(i)If the PED<1, when price falls, total
expenditure on food falls
(ii)If there is a subsidy, there will be a lower
cost of production and hence price will fall.

SRJC

What it should be
The point is people are less able to buy same amount
with a lower income.
The analysis, is ceteris paribus, price remaining
unchanged, people are now able to buy less goods
with lower income. Hence at the same price, the
demand falls, causing shift to the left.
Normal goods are goods which
(i)
follows the downward sloping demand
curve where ceteris paribus, price fall
will lead to an increase in quantity
demanded for the good
(ii)
has positive income elasticity, meaning it
is positively related to income changes.
Hence, if income rise, demand rise., If
income falls, demand will fall.
Normal goods may be necessity or luxury depending
on the degree of change when one of these demand
factors changes, ceteris paribus.
Necessities by definition have a low positive income
elasticity of demand and a low price elasticity of
demand.
Luxury items have a high positive income elasticity of
demand. They also have a high price elasticity of
demand.
Same problem as qn 1:
Merely stating a relationship is not an explanation:
For such a technical question, the whole demand and
supply framework must be competently and rigorously
demonstrated
In such a framework, the ceteris paribus is important
and must be qualified clearly what these other
relevant/pertinent factors are in the analysis.

18
There is also lack of clear competence in explanation
price changes and non-price changes
(III) Quality of answers
The length of a good attempt was 4.5 sides for this question.
(IV) Organisation of answer
It would be easier to first apply each of the 2 triggers at a time and then comment on the net effect of both
triggers on each good. Thereafter, make the comparison of the impact on the total expenditure between the two
goods.

SRJC

19
3

(a)

Explain how market dominance and imperfect information can lead to market
failure.
[10]

(b)

Discuss the view that government intervention in such markets would always
lead to a better outcome for society.
[15]

Suggested answer for part (a)


Introduction
Definition: Market failure occurs when the operation of the market fails to achieve economic
efficiency in the allocation of scarce resources i.e. it is possible to change the existing
resource allocation to make someone better off without making someone else worse off.
There are a few causes of market failure including market dominance and imperfect
information. Market dominance implies that a large portion of the market share is controlled
by one or a few large firms; this can be seen in market structures such as the monopoly
market structure and the oligopoly market structure. When one side of the market (either
buyers or sellers) has better information than the other, imperfect information exists.
Body Market dominance leads to market failure
P: Market dominance could lead to market failure as there is allocative inefficiency.
E: In the imperfect market structure, firms can enjoy market dominance. Such firms are price
setters and thus, their demand curve is downward sloping. At the profit-maximising level of
output, the price charged is greater than marginal cost. With the supernormal profits earned
by such dominant firms, other producers will be enticed to enter the industry. But entry
barriers are so high that the dominant firms shut out all competition and continue to produce
the output level where his price is greater than the marginal cost of production.
E: For example, the profit-maximising monopolist, by virtue of having a high degree of
market power, restricts his output and charges a higher price for his product than socially
desirable. Thus, the monopoly results in an inefficient outcome for the society.
Price, Revenue, Cost
A

MC
B

Pm
Pc

Ec

Em

AR

X
0

MR
Qm Qc

Quantity

Comparison of Equilibrium of Monopoly with Perfect Competition (assuming


same cost conditions)
The socially ideal price and output are 0Pc and 0Qc respectively where P=MC. This means
that the value the society places on last unit of output produced is exactly equal to the
opportunity cost incurred by the society in having that last unit. For that output level, the
total benefit to the society is measured by the area 0AEcQc while the total cost is area
SRJC

20
0XEcQc. Thus total net benefit to society is area AEcX. Given his profit-maximising motive,
the monopolist will produce where MC = MR and MC is rising. At output, 0Qm, the price
charged by the firm is greater than his marginal cost of production (P>MC). This means that
society values the last unit of this good more than the alternative goods forgone (i.e.
opportunity cost). Therefore, society would benefit from having more of this good. By
restricting output to 0Qm, and charging a price of 0Pm, the total benefit is area 0ABQm but the
total cost is area 0XEmQm. Hence the total net benefit to society falls to area ABEmX.
L: Hence, the monopolists profit-maximising behaviour results in a deadweight loss to
society of area BEcEm. In this case, market dominance results in allocative inefficiency and
market failure.
P: Market dominance could lead to market failure as there is X-inefficiency.
E: The dominant firm may incur higher costs of production due to X-inefficiency, a term
coined by Prof. Harvey Lieberstein, to mean organisational slack that exists in firms where
managers do not strive to keep costs as low as possible. Being the dominant producer and
with high barriers to entry, there is no competitive pressure on profits. Thus, cost control can
become lax. There may be overstaffing and less effort to keep technology up-to-date, hence
resulting in higher average cost of production than it would otherwise be. In this case, the
firm would be producing at a point above the long-run average cost curve.
L: Thus, market dominance results in X-inefficiency and market failure.
Body Imperfect information leads to market failure
P: Similarly imperfect information could also lead to market failure.
E: A particular case of imperfect information is that of asymmetric information whereby one
party has significantly more information about a good or service than the other side of the
economic transaction. This gives rise to sub-optimal outcomes in resource allocation.
E: For example, in the healthcare market, patients often know much less about their medical
conditions or treatments available compared to doctors who generally have better
information regarding medical procedures and treatment. On the other hand, buyers of
health insurance may not accurately declare their own state of health to insurance
companies. Asymmetric information presents an opportunity for the party with more
information to exploit the information gap, resulting in sub-optimal outcomes. Another
example is the second hand car market in which the seller of cars is likely to know more
about the quality of the cars being sold. In contrast a buyer of the same second hand car is
unlikely to be able to determine the quality of the product effectively. The rational buyer
would then assume that the car is likely to be of an average quality and will not be willing to
offer a higher price even if the car is actually of a better quality. Thus, sellers of good quality
cars will not be willing to accept a below average price and will leave the used car market
while sellers of below average and average quality cars will remain. In the end, the market
for used cars will be dominated by cars of average or below average quality and buyers will
either be over-paying for cars (in which case price would be greater than marginal cost) or
will leave the market entirely.
L: Hence, imperfect information could lead to market failure.
Note: Can also accept lack of information on external cost or external benefit. Thereafter,
measures discussed in part (b) must explain how this problem is addressed.
Conclusion
Given these sources of market failure, the government may need to step in and intervene to
bring about a better outcome to society.

SRJC

21
Level
L3

Descriptors
Candidate is able to explain clearly, with appropriate examples, why market
dominance and imperfect information would lead to market failure.
Candidate makes some attempt to explain why market dominance and/or imperfect
information would lead to market failure. Some conceptual mistakes in explanation.
Smattering of points, glaring conceptual errors in explanation, no attempt to link
imperfect information and market dominance with market failure.

L2
L1

(b)

Discuss the view that government intervention in such markets would always
lead to a better outcome for society.
[15]

Introduction
Government may intervene in markets with excessive market dominance or imperfect
information. Direction: This is done to bring about a better outcome to society, i.e. trying to
achieve allocative efficiency, productive efficiency and higher consumer welfare.
Body 1 (Thesis 1): Explain how MC pricing would achieve allocative efficiency (better
societal outcome)
P: In the case of market dominance, the government may use marginal cost pricing to
achieve allocative efficiency.
E: For example, given a case of a natural monopoly, the government may force the
monopoly to set prices at marginal cost. A natural monopoly is a firm that has such large
economies of scale such that a single firm is able to supply the entire market at a price which
is lower than two or more firms.
As shown in the diagram below, initially the firm would set quantity where marginal cost is
equal to marginal revenue and the price would be P. Clearly the initial price is much greater
than the marginal cost of production, meaning that the consumers value the last unit of the
good much more than the opportunity cost of resources used to produce the good. When the
government intervenes, it forces the monopoly to set price equal to marginal cost of
production, i.e. Pm.
Price, Revenue, Cost

P
X

P0
Pm
0

Y MC
AR

A
Q

MR

Qm

AC

Quantity

L: This increases consumer welfare as consumers are able to obtain more of the good at a
lower price, and achieves the microeconomic goal of allocative efficiency.

SRJC

22
Body 2 (Anti-Thesis 1): Evaluate MC pricing
P: However, with marginal cost pricing the firm would make subnormal profits in the long run.
E: With reference to the figure above, the firm would earn a total revenue (price times
quantity exchanged) of area PmYQm0. The total cost of production would be P0XQm0. This
means that the firm would earn a subnormal profit of P0XYPm. The firm will not be able to
continue production in the long run and would have to shut down.
The government may need to subsidise the firm in order for the firm to continue producing in
the long run. This represents an opportunity cost to society as the resources employed for
subsidies could have been used for other purposes.
L: Thus, government intervention may actually result in a worse outcome for society.
Body 3 (Thesis 2): State ownership could lead to a better societal outcome.
P: The state takes over the production of the good completely in order to protect the
interests of the consumers.
E: Freed from the profit motive, there should be no tendency for the state-owned firms to
make high profits. As a result, the price charged would fall. This would reduce the difference
between price and marginal cost, leading to a fall in allocative inefficiency. If profits are
made, these could be passed on to the public in the form of lower future prices or in the form
of tax relief / rebate.
L: Hence, state ownership would lead to a better outcome for society.
Body 4 (Anti-thesis 2): Government failure would lead to a worse outcome for society.
P: However, state enterprises tend to suffer from a higher level of inefficiency in production,
e.g. red tape that refers to excessive formality and routine required before official action can
be taken is involved.
E: Run by the state, there is little incentive for efficiency and innovation as there is no profit
motive. Additionally, bureaucracy and red-tape can lead to slower decision-making, which
can translate into higher cost of production. As a result, consumers would end up paying a
higher price for the good. In addition, once the profit motive of the private sector is removed,
the question remains as to what may motivate those in charge of providing the good. There
is no entirely clear answer to this question. At its worst, political objectives that some
government officials have may lead to inefficiency in resource allocation. For example, they
may only provide the good to people who support their political parties.
L: As a result, the government may create inefficiencies when they intervene in the market.
Hence, government failure arises, which would lead to a worse outcome for society.
Body 5 (Thesis 3): Government intervention to correct imperfect information
P: For markets with clear asymmetric information problems, for example, the healthcare
industry, the government may introduce regulation to bridge the information gap.
E/E: To reduce asymmetric information in the healthcare market, a government can force the
hospitals to be more transparent in their operations. For example, the hospitals have to be
transparent about the cost of medicine, the doctor fee, the cost of equipment to prevent
patients to be charged excessively by the unethical doctors. This can be done through
legislation just like Singapores hospitals where healthcare providers are expected to clearly
display their prices for various procedures and are expected to abide by these prices.

SRJC

23
L: This prevents healthcare providers from exploiting consumers by charging a price more
than marginal cost as all patients have the same information.
Body 6 (Anti-thesis 3): Evaluation of government intervention to solve imperfect
information
The legislation is straightforward to understand and relatively easy to monitor, e.g. check
hospitals websites and posing as consumers to verify information stated on websites.
However, the government itself faces the problem of imperfect information, and is often
unable to ascertain the private costs and benefits of individuals and firms. For example, the
government may lack the specialised medical knowledge necessary to regulate the
healthcare industry; simply getting healthcare providers to display their prices may not totally
close the information gap between consumers and providers.
Conclusion
Thus, government intervention may not always bring about a better outcome to society. This
is because any attempt by the government to intervene in markets may lead to undesirable
side effects. Furthermore, the government may create inefficiencies when they intervene in
these markets due to factors such as administrative costs and lack of information. All this
can lead to government failure. Interestingly, the market itself may be able to solve the
problem of asymmetric information. For example, there are firms such as Vicom (inspects
used automobiles), which consumers can use to find out more about the quality of the preowned products that they wish to purchase. Overall, a market-based solution which
preserves consumer sovereignty would be the preferred choice in the long run.
Level
L4

L3

L2
L1

SRJC

Descriptor
Candidate is able to provide a thorough explanation and evaluation of the
policies that the government can use to correct the stated sources of market
failure.
In addition, the candidate makes competent use of examples to support the
analysis and is able to provide a reasoned conclusion as to whether
government intervention would lead to a better outcome to society.
Candidate is able to explain and evaluate clearly, with some examples, the
policies that the government can use to correct the problems of market
dominance and imperfect information.
For competent explanation and evaluation of policies that do not address the
examples that are brought up in part (a) max. middle band
Candidate makes some attempt to explain and evaluate the policies that the
government can use to correct the problems of market dominance and/or imperfect
information.
Smattering of points, glaring conceptual errors in explanation.

24
Examiners comments:
3
(a)
Explain how market dominance and imperfect information can lead to market
failure.
[10]
Strengths:
1. In general, candidates could explain how market dominance can lead to allocative inefficiency using a
diagram.
2. In general, candidates could explain an example on the health care market to illustrate imperfect
information causing market failure.
Question interpretation:
1. Some candidates explained how imperfect information caused market failure in the following way:
Consumers do not have perfect knowledge of how prices are determined. Potential competitors do not
have perfect knowledge of the technology or skills that are required to produce and sell the product. Hence,
dominant firm can set prices to maximise profits. But this is still related to market dominance! Imperfect
information needs to be interpreted as asymmetric information, lack of understanding of the true costs or
benefits of a product or service, complex information and/or inaccurate or misleading information.
2. Some candidates were not able to clearly link market dominance with market failure. Market dominance is
related to the firm's control over a large proportion of the market share, which allows the firm to be a price
setter. The firm is able to restrict quantity sold in order to increase prices, thus, giving rise to a downward
sloping demand curve.
3. A minority of candidates used the example of government underestimating the size of the marginal external
cost when taxing demerit goods to explain imperfect information. This is not a good example to use as the
question specifically asks you to explain market failure. The above example is actually an instance of
government failure.
Clarity and precision:
What you wrote

What it should be

P>MC: Value that society places on the good is more


than the opportunity cost incurred in producing the
good.

P>MC: Value that society places on the last unit /


additional unit of the good is more than the
opportunity cost incurred in producing that last /
additional unit of the good.

Externalities as a type of imperfect information is


acceptable if candidates make an explicit link between
externalities and imperfect information.
As consumers do not take into account the social cost
of consuming cigarettes, there is overconsumption of
cigarettes.

There is a lack of understanding of the true costs of


consuming cigarettes as consumers may not know the
cost that they inflict on a third party. (Go on to
explain the MEC of consuming cigarettes.) Hence,
they underestimate the true cost of consuming
cigarettes, leading to overconsumption of cigarettes.

When price of the good exceeds the marginal cost of


production, the consumer welfare is not maximised.

When the price of the good exceeds its marginal cost


of production, the sum of producer and consumer
welfare is not maximised.

As there is an underproduction of the good, there is a


deadweight loss incurred.

Each additional unit produced between the market


equilibrium output and the socially desirable output
will add more to social benefit than social cost. The
summation of this excess provides a monetary value of
the welfare loss to society(deadweight loss) that
occurs due to the underproduction.

SRJC

25
(b)

Discuss the view that government intervention in such markets would always lead to a better outcome
for society.
[15]

Question interpretation:
1. Candidates should state clearly what they understand by a better outcome for society, e.g. a fall in
economic inefficiency and a rise in consumer welfare.
2. Some candidates narrowly defined better outcome as just a rise in consumer welfare and went on to
explain how government intervention could lead to a fall in price and improvement in quality of the
product. This scope is too narrow. It is important to discuss the impact of government intervention on
efficiency in resource allocation and/or equity.
3. Government intervention in such markets - this means that candidates have to discuss the measures that will
address the sources of market failure that are highlighted in (a). E.g. some candidates explained asymmetric
information in (a) but discussed measures to correct negative externalities in (b)!
4. Candidates should introduce policies that directly address the root causes of market failure, as discussed in
part (a). For example, some candidates introduced subsidies to solve the problem of market dominance while this policy may reduce the final prices of products sold to consumers, it would not solve the problem
of inefficiency in resource allocation. In fact, the misallocation of resources may be even worse!
Rigour in analysis:
What you wrote

What it should be

When production is undertaken by the state, the Once the profit motive of the private sector is removed
government may use the product for political when the state takes over the production, the question
purposes.
remains as to what may motivate those in charge of
providing the good. Political objectives that some
government officials have may lead to inefficiency in
resource allocation. For example, they may only
provide the good to people who support their political
parties. In this case, the misallocation of resources by
the state leads to government failure.
When production is undertaken by the state, the
government would charge a price that is equal to MC.

When production is undertaken by the state, there is no


tendency for the government to maximise profits.
Rather, the government would want to maximise the
welfare of the society. Hence, it would charge a price
that is equal to MC, which maximises societys
welfare since allocative efficiency is attained.

The government could increase competitiveness in the


industry by introducing new firms into the industry.

The government could deregulate the industry by


lowering the barriers to entry. New firms would be
attracted by the possibility of earning supernormal
profits and enter the industry easily, thus increasing
the level of competition in the industry.

When the government deregulates the industry, there When the government deregulates the industry, there
is more competition. Hence, there would be a rise in is more competition for the incumbents. Hence, there
allocative efficiency.
could be a fall in the demand for the incumbents
goods as well as a rise in the value of the price
elasticity of demand.

SRJC

26

Assuming that the demand curve for one of the


incumbents shifts to the left and becomes more price
elastic as represented by AR1, there is a fall in
equilibrium price to Pc. Assuming that there is no
change in the equilibrium quantity, the difference
between price and MC is now smaller. As a result,
there is a smaller deadweight loss of the area BEmEc.
What all this means is that government intervention
can reduce allocative inefficiency, leading to a better
outcome for the society.
The government could provide more information on
health procedures which would solve the problem of
underconsumption.

SRJC

The government could provide more information on


health procedures to the consumer, who would then be
able to accurately measure his own private benefits
obtained from a particular medical treatment. This
would reduce the information gap between producers
and consumers and allow consumers to pay a price
which is closer to the marginal cost of production,
reducing the extent of resource misallocation.

27
4

Discuss the extent to which being a small and open economy affects the policy
decisions to achieve price stability and full employment in Singapore.
[25]

Introduction
Singapore has a small population of about 5 million people as well as a small geographical
area of only 704 km sq. This disadvantaged her in terms of the availability of resources and
size of domestic market. Due to these constraints, the Singapore economy has to be open
which means that the country is highly dependent on trade, foreign investments as well as
labour. Exports and imports are about 400% of GDP and FDI is 70 % of total investments.
Direction: Being small and open would mean that price instability and unemployment are
caused mainly by external factors and in addition they also have bearings on the
effectiveness of macroeconomic policies. Policy makers must therefore take these two
characteristics into account in deciding which policies to use. However, there are also other
factors policy makers must consider such as other causes of price instability and
unemployment, whether these problems occur singly or together, the time period, the nature
of the policies and policy conflicts.
Body
P: Being small and open means that the main type of inflation is often that of importprice-push inflation or/& external demand led demand pull inflation and thus the use
of exchange rate policy of modest and gradual appreciation is often the main policy
tool to achieve short to medium term price stability in Singapore.
E/E: Being open make Singapore most vulnerable to supply shocks such as increase in the
price of oil and other raw materials cause by the booming economies of China and India,
higher prices of food due to bad weather, and higher foreign inflation. Increase in domestic
inflation comes about through higher prices of imported final products as well as through
higher cost of imported inputs which raise cost of production which in turn is translated into
higher domestic prices.
General price level

AS1
AS0

P1
P0

AD
0

Figure 1

Y0 Y1

Real national output

As a result of higher cost of production due to higher import prices, ceteris paribus, profits
fall and producers cut back on production causing the AS curve shifts to the left from AS0 to
AS1. This results in shortages and causing upward pressure on prices and prices increase
from P0 to P1.
Since the root cause of the inflation is higher import prices, exchange rate policy would be
an appropriate choice. To fight such inflation, the Singapore government can allow the

SRJC

28
Singapore dollar to appreciate. An appreciation of the SGD means that one unit of the
domestic currency can exchange for more units of the foreign currency. This lowers the
prices of imported final goods and services in SGD. This immediately reduces the rate of
inflation. Prices of imported inputs and raw materials in SGD will also be lowered which in
turn lower the firms cost of production. Ceteris paribus, this increases profits and firms
increase production and the AS curve shifts back to the right again, hence lowering the rate
of inflation as prices fall from P1 to P0.
L: Hence, an appreciation of the Singapore dollar effectively reduces imported price push
inflation as it directly targets at the root cause of the problem- high import prices.
P: Being small and open, Singapore is also very prone to demand pull inflation and so
exchange rate policy can also be used to influence net exports.
E/E: Firstly, when the world economy booms, Singapore will enjoy higher net exports as
rising world incomes will encourage greater consumption which benefits Singapores export
sector. Also, Singapore has been successful in developing her comparative advantage in the
production of goods and services such as electronics, chemicals, pharmaceuticals as well as
services such as tourism and finance. This too boosts her exports. Assuming demand for
imports does not increase by as much, net exports increase.
Ceteris paribus, AD increases sharply as net exports is a big component of AD as explained
earlier. The fact that Singapore is a small economy, large increase in AD from AD0 to AD1
will easily move the AD curve nearer to the full employment level of output, Yf, as her
resources are limited. Ceteris paribus, this will cause shortages as the economy faces
capacity constraints in the short run due to insufficient labour and capital goods. Costs will
then increase which will eventually cause prices to increase from P0 to P1.
General price level
AS
P1

E1

P0

AD1

E0
AD0
0

Y0

Yf

Real national output

Figure 2
Once again, exchange rate policy in the form of appreciation can be considered as it will
target the root caus, which is net exports, to reduce the rise in AD. When the Singapore
dollar appreciates, its exports will be more expensive in foreign currency. The rise in export
prices will lead to a fall in its quantity demanded by foreigners, ceteris paribus. At the same
time, imports would be cheaper in domestic currency. Hence, the quantity demanded for
imports will rise, ceteris paribus. Assuming that the sum of price elasticity of demand for the
countrys exports and imports is greater than one or that the Marshall-Lerner condition is
met, the currency appreciation will lead to a fall in Singapores net exports. This will shift the
AD curve from AD1 to AD0 and this will cause prices to fall from P1 to P0.

SRJC

29
L: Hence, exchange rate policy of appreciation can achieve price stability in Singapore.
Evaluation (anti-thesis):
However there are considerations other than being small and open that affects policy
decisions that control inflation.
Although a strong Singapore dollar may be effective in maintaining a low rate of inflation, it
also exerts a contractionary effect on real national output and hence, economic growth and
employment. Figure 2 shows that when AD falls, producers cut back on production from Yf to
Y0 and reduce the number of workers employed. Thus in the short run, the government may
face with the problem of achieving lower inflation at the expense of increased
unemployment. Policy makers must weigh the benefit of such a policy especially during
times of weak global demand. Thus exchange rate policy alone may not be sufficient to
achieve both goals of price stability and full employment.
To deal with demand pull inflation, the government has at its disposal other demand
management policies such as monetary and fiscal policies.
P: As Singapore is open to capital flows, monetary policy is not effective in achieving
price stability and full employment in Singapore.
E/E:
 A contractionary monetary policy may be used to reduce the rate of inflation. The
government reduces the money supply to bring down the rate of inflation. When money
supply falls, interest rates will go up, increasing the cost of borrowing. Hence, firms and
consumers will reduce their quantity demanded for funds for investment and
consumption purposes respectively. These will result in a fall in investment and
consumption, reducing aggregate demand, ceteris paribus, and the general price level.
Vice versa, an expansionary monetary policy can be used to increase AD and thus
production and employment, thus achieving full employment.
 In the case of Singapore, using interest rates to affect the level of economic activity is of
limited effectiveness due to Singapores small size and its high degree of openness to
trade and capital flows. Because of Singapores role as an international financial centre,
the economy is very open to capital flows. As a result, small changes in the difference
between domestic and foreign interest rates can lead to large and quick movements of
capital. This makes it difficult to target money supply in Singapore.
L: Thus being open means that Singapore does not use interest rate as the monetary tool,
whether to achieve price stability or full employment in the economy.
P: Having a small domestic market due to the small population in Singapore also
reduces the effectiveness of fiscal policy in achieving price stability.
E/E: To reduce the inflation rate, the government can cut back on its own expenditure.
Since government expenditure is a component of aggregate demand, this action will
immediately reduce aggregate demand.

The fall in aggregate demand is illustrated by a leftward shift in the aggregate demand
curve, from AD1 to AD0. At P1, aggregate supply exceeds aggregate demand. The
resulting surpluses throughout the economy would drive down prices. As a result, firms
would produce less: there would be a movement down along the AS curve. At the same
time, the decrease in prices would raise the level of aggregate demand: that is, there
would also be a movement down along the AD curve. The surpluses would be eliminated
when the general price level has fallen to P0. At the new equilibrium point E1, inflation is
reduced.

Likewise, government can increase G and reduce taxes if the objective is to achieve full
employment.

SRJC

30

However the small domestic market relative to external demand means that the
reduction or increase in domestic demand via the use of fiscal policy will only affect AD
by a limited extent as compared to a policy instrument that targets external demand
such as the exchange rate policy.

L: Thus fiscal policy is often not the anchor policy that is used to achieve price stability or full
employment but plays more of a supporting role due to the small and openness of the
economy.
Being small and open has also contributed to a large extent the priority given to the use of
supply side policies in Singapore to achieve both the goals of price stability and full
employment. Supply side policies can complement exchange rate policies in increasing net
exports without having to sacrifice the goal of full employment. Being small means facing
capacity constraints quickly and being an open economy means that there can be large
increase in AD. A combination of both would surely fuel demand pull inflation. The solution is
to use SS side policies to increase productive capacity. SS side policies can also deal with
inflation that is caused by domestic price pressures such as falling productivity. In the
context of Singapore where full employment has been achieved for many years, the need to
maintain long term price stability for sustainable growth is also a consideration for policy
makers.
P: Being small and open, supply side policies would be able to achieve not only price
stability due to increase in AD but also increase in employment caused by both fall in
AD as well as rapid changes in the structure of the economy.
E/E: Some examples of supply-side policies include:
a. Market-oriented supply-side policies such as
implementing policies to restrict the activities of trade unions so that they will not ask
for unreasonable wage increases. Trade union reform may also increase workers
flexibility and mobility and cut down on the number of days lost through strikes.
b. Interventionist supply-side policies such as
granting subsidies and establishing more training centres and courses to encourage
education and training to increase labour productivity.
giving tax incentives to firms and investment grants to encourage research and
development in product development and production techniques.
General price level

P0

AS0

AS1

E0
E1

P1

AD

Y0YF0Y1 YF1 Real national output

Except for the crisis years like 2003 and 2009, Singapore was at operating close to full
employment as one can see that the AD curve cuts at the vertical section of the AS curve.

SRJC

31
There is therefore a high risk of demand pull inflation due to a lack of capacity to increase
output as our labour force is small.
But with the use of supply side policies, land is reclaimed from the sea, transport network
widens, number of industrial sites increase and so does labour productivity. This increase in
productive capacity enables firms to increase output to meet the rising AD during economic
booms without firms having to incur higher costs.
It works just as well during the above crisis years when AD fall or when the use of
appreciation causes AD to fall as explained at the beginning of this essay. For an open
economy like Singapore, the fall in AD is largely due to fall in external demand as a result of
our major trading partners suffering from recession. The use of supply side helps to contain
inflation by means of increasing productivity. This helps to make our exports more
competitive compared to our rivals. In the case of rising export prices due to appreciation of
the SGD, it helps to further reduce cost of production brought about by cheaper imports and
together, this can reduce the rise in price of exports brought on by the appreciation of the
SGD.
When the AS increases, the AS curve will shift to the right from AS0 to AS1. As a result, the
general price level falls from P0 to P1 while the real national output increases from Y0 to Y1.
This increase in production would lead to a rise in the derived demand for labour. Hence,
unemployment falls. There is also an increase in the full-employment level of national output
(or potential growth) from YF0 to Y F1.
L: Thus there are other factors such as conflicts in economic goals that influence the choice
of the governments policy to achieve price stability.
P: The root cause of cyclical unemployment tends to be a fall in external demand (e.g.
due to global economic recession) for a small and open economy  in achieving full
employment, being small and open sees the use of depreciation to target export
demand to reduce cyclical unemployment. On top of that, the small domestic demand
means that the use of fiscal policy and monetary policy would not be very effective.
This requires more targeted measures to reduce cyclical unemployment e.g. the use
of job credit scheme.
E/E:
Government may allow the currency to depreciate to increase (X-M). Since net exports
increase, AE also increases, ceteris paribus. The rise in AE would bring about a shortage of
goods and services. Hence, there is an incentive for firms to increase production, leading to
a multiple increase in national output, national income and employment, thus reducing
cyclical unemployment.
The lack of effective policy tools for a small and open economy like Singapore often requires
the government to move beyond the conventional policy instruments and invent new and
more effective ones. For example, in 2009, the Jobs Credit Scheme was introduced to
encourage businesses to preserve jobs in the downturn. Businesses receive a cash grant
based on the Central Provident Fund (CPF) contributions they have made for their existing
employees. It provides a significant incentive for businesses to retain existing employees,
and where their business warrants, to employ new ones. This is a temporary one-year
scheme which benefits about 120,000 employers and hence prevents escalation of cyclical
unemployment.
L: Thus being a small and open economy does influence policy decisions to achieve full
employment in Singapore to a large extent.

SRJC

32
Evaluation (anti-thesis): However there are other important considerations such as
the global economic sentiment.
1) If the global economic sentiment is pessimistic such that consumers around the world
are not responsive to changes in prices, depreciation will not bring about an increase in
(X-M) and thus the required AD to generate employment opportunities. Thus,
government would need to consider other supplementary policy options to reduce
cyclical unemployment. The same applies if the other countries also depreciate their
currency amidst global recession, reducing the effect of Singapores depreciation on her
export competitiveness and thus growth and employment.
2) Also, if structural unemployment is what the government is trying to reduce, supply side
policies to ensure that our workers are constantly equipped with the necessary skills for
the knowledge-based economy would be the preferred option, regardless of whether the
economy is a small and open one or large and relatively closed.
P: Being small and open however does mean that controlling inflation could take
priority over the reduction of unemployment and that could shape policy decisions of
Singapore government in the event of a stagflation.
E/E: Low and stable prices could increase her export competitiveness which is crucial
avenue to generate income and employment given the constraint of the small domestic
market. Also, the stable economic environment brought about by price stability would instil
confidence in the economy and that would bring about an increase in FDI (70% of total
investment in Singapore) and thus greater job creation. Being small means that Singapore is
highly dependent on external demand (70% of total demand) to generate income and
employment in the economy.
L: Maintaining price stability could help attain both the goals of internal balance and full
employment for a small and open economy and thus policy choices could revolve around
reducing inflation during a stagflation.
Conclusion:
Being small and therefore the need to be open has provided many constraints for the
Singapore government in its use of policies. Often, the cause of the problem is affected by
the fact that Singapore is small and open. Although it is an important consideration, there are
other factors that the policy makers need to take into consideration for sound policy
decisions e.g. internal and external economic condition as well as the citizens receptiveness
to the policies. The multi-causal and interconnectedness of all the macroeconomic elements
is possibly the reason why economists and policy makers often have different opinions and
thus difficulties in deciphering which are the best policies to achieve the various
macroeconomic goals.
Level
Descriptors
L5
Answer has to demonstrate strong and accurate analysis, good evaluation and well
synthesized judgment, showing strong awareness of the limitations faced by the
Singapore government on achieving the various macroeconomic goals for a small
and open economy.
L4
For an answer that demonstrates clearly how being small and open
constrains the policy options of Singapore;
results in exchange rate and supply-side policies being the dominant policies
used to control inflation;
results in the need for more targeted approach to reduce cyclical unemployment.
Answer has to be balanced with the awareness that other factors will affect the
choice of policies in Singapore
L3
2 sided answer with strong analysis but limited scope and lack synthesized

SRJC

33

L2

L1

SRJC

judgement
One-sided answer without consideration of other factors max. lower band
Some analysis with some attempts to link policies to price stability and full
employment.
Only link to price stability or full employment max. middle band
Descriptive knowledge of policies, which does not address question max. upper
band

34
Examiners comments
4
Discuss the extent to which being a small and open economy affects the policy decisions to achieve
price stability and full employment in Singapore.
[25]
Strengths:
1. Most candidates have good knowledge of the different types of policies and their limitations.
2. Some candidates answers were of high quality showing good understanding of question requirements and
responding to it in a mature and sensible manner. Credit should be given to them for
Weaknesses:
1. Candidates could have used their knowledge better to apply to what was asked rather than to regard it as
just a question asking for them to explain how the policies work and their limitations. They should avoid
starting every paragraph with this kind of sentence: The next policy is contractionary fiscal policy.
2. Candidates could do with greater flexibility in the use of the knowledge that they have. For example, they
should not be repeating the multiplier process for monetary, fiscal and even exchange rate policy. Though
the multiplier process was explained by almost every student, it did not come through that they understood
it well. Most mentioned about the increase in income but said nothing about employment which was what
the question was asking. Limitations of policies were likewise memorised and not elaborated now that they
are given a new context that is different from what they have done in their tutorials or tests.
3. Candidates answers lack appropriate structure to manage the complexities of the question.
4. Candidates often got carried away with their description of how the policies work that they forgot what the
end point of the question was.
Question Interpretation:
1. The meaning of small and open must be explained clearly and linked to the causes of price instability and
unemployment.
2. Policy recommendations must be linked to the root cause identified and they should avoid prescribing a
series of policies independent of the cause of the economic problem.
3. Lastly, the question says to what extent and so there is a need to consider situations where other factors
play a part in influencing the choice of policies. Candidates probably did not fully understood the question.
For eg when they identified the problem of conflicting objectives, they simply took it as just one of the
limitations of policies without highlighting it to the reader that this is a consideration in the choice of
policies. Probably they were not aware that this was a factor that policy makers need to take into account or
they did not know how to tweak it to fit the question requirement.
Clarity and precision:
What you wrote

What it should be

Full employment is at Yf

The full employment level of output is at Yf.

Singapore can adopt expansionary ERP.


(There is no such thing as expansionary ERP)

Singapore can adopt exchange rate policy of allowing


the Singapore dollar to appreciate in the face of a
worldwide economic boom.

. multiple increase in Y, output and employment


thus achieving full employment

The increase in government expenditure will result in


a multiple increase income and output. This can only
be achieved when firms increase its employment of
workers leading to a fall in the unemployment rate and
if the increase in AD is high enough, the economy
may achieve full employment.

Due to Sp being an open ecy, it affects the policy


decision for using MP to a large extent.

Due to Sp being small and open, monetary policy is


not very effective and therefore it is not used to
influence AD.

Net X forms 30% of AD therefore net X increase and


AD increases.

Net exports is 30% of AD and therefore increase in net


exports will lead to a large increase in AD.

Rigour in analysis:

SRJC

35

What you wrote

What it should be

G is a small % of GDP therefore increase in G would


not be expansionary enough.

G is a small % of GDP therefore increase in G would


not be expansionary enough to offset the fall in AD
brought about by the large fall in net exports.

. MPM is high therefore FP not effective.

A high MPM means that much of the income


generated from the increase in government spending is
leaked out from the circular flow of income in the
form of expenditure on imports. As a result domestic
firms experience smaller increase in demand and so
there will be a smaller increase in employment of
factors of production.

Contractionary MP is not effective because of large


capital flows and therefore it is difficult to target
money supply.

Contractionary MP is not effective in reducing money


supply because the rise in interest rates will result in
large increase in short term capital inflow due to the
openness of the Singapore economy. Banks are then
faced with huge amount of funds causing money
supply to increase which offsets the initial fall in
money supply.

training increase productive capacity and hence


achieve low unemployment and low inflation if the
economy is at full employment

When workers are given more training to acquire new


skills, their productivity increases. Each worker can
produce more goods then before. The economys
productive capacity increases and so it is better able to
accommodate the rising AD and therefore avoiding
demand pull inflation.
At the same time, with new skills, structural
unemployment will be reduced as the unemployed are
better equipped with skills to find jobs in the new
industries.

SRJC

36
5

Economists forecast a world trade growth of 9.5% for 2010, with developing
countries' trade growing by 11% and developed countries' trade growing by 7.5%.
(a)

Explain why it is important for a country to attain its macroeconomic goals.


[10]

(b)

Discuss whether developing countries have more to gain from globalisation


than developed countries.
[15]

Suggested Answers to (a)


I.
1.

Introduction
Macroeconomic goals are often asserted to refer to sustained economic growth, low
unemployment rate, price stability and satisfactory balance of payments. Direction of
essay: The attainment of these macroeconomic objectives is important to any
economy as there are benefits from attaining these goals, e.g. achievement of the
respective macroeconomic objective can lead to the attainment of other economic
objectives.

II.
2.

Body
Economic growth takes place when there is an increase in the countrys real GNP.
Economic growth can refer to actual growth or potential growth. Actual growth is the
increase in national output actually produced due to the increase in aggregate
demand. On the other hand, potential growth refers to the increase in the fullemployment level of national output due to the increase in aggregate supply.
Benefits of economic growth:
 P: Attaining economic growth is important because it increases the standard of
living.
E: Economic growth means that national income increases. Ceteris paribus, this
means that peoples purchasing power has increased. Hence, they are able to buy and
enjoy more goods and services, leading to a rise in their standard of living. Economic
growth means that government can collect more tax revenues to finance expenditure
on infrastructure, education and health care. All these will mean that the citizens are
able to enjoy a higher level of economic welfare.
L: Hence, a country wants to attain economic growth.
 P: Attaining economic growth is also important because it brings about greater
equality in income distribution.
E/E: Where there is economic growth, there will be an increase in government tax
revenues. This makes it easier for government to redistribute income to the poor,
without the need to raise tax rates. This is especially crucial in Singapore as our
economy is increasingly facing the issue of income inequality, which has every
tendency to exacerbate with globalisation and rapid technological advances. These tax
revenues can be redistributed to the poor in the form of subsidies, hence narrowing the
income gap. For example, because of the good economic performance in 2010 and
the positive economic outlook, the Singapore Government distributed Growth
Dividends to all adult citizens in 2011, with the lower income groups receiving more.
L: Thus, a country wants to attain economic growth.

3.

Low inflation represents price stability and creates confidence in people within the
economy. It creates a more certain environment in which both businesses and
households can plan and operate more efficiently. It is an essential ingredient for
sustainable growth in investment, output and jobs.

SRJC

37
Benefits of low inflation
 P: Low inflation or price stability promotes investment and saving, and hence
can have a positive effect on economic growth and employment.
E: With low inflation, firms are better able to predict future revenue, costs and thus
profits with greater certainty. Foreign investors may also be attracted to invest in the
country due to relatively lower costs compared to other countries with higher inflation
rates. Moreover, low inflation serves as incentives for people to save as real value of
money remains constant. If nominal interest rate is more than inflation rate, the
positive real interest returns will encourage people to save. With increased savings,
there will be more funds for investment. Since inflation is low, this means that the
economy is not at full employment and thus increase in investment will lead to
increase in aggregate demand, thus hence increasing national output, employment
and a multiple increase in national income. This allows the country to achieve two
macroeconomic objectives. In addition, in the long run, the increased investment on
capital goods will increase the productive capacity of the country, hence increasing
long term potential growth and standard of living.
L: Hence, it is important for a country to attain low inflation.
 P: Low inflation can also have a positive effect on balance of payments.
E: With low inflation relative to other countries, this allows a country to enjoy export
competitiveness, where exports become relatively cheaper. This is especially crucial
for Singapore, a highly export-oriented economy. With relatively cheaper exports, it will
lead to an increase in the demand for exports. Hence total revenue from exports will
increase. Assuming total expenditure from imports to be constant, there is a net
exports surplus, leading to a current account surplus. In addition, low inflation which
attracts foreign investors investing in the country will also lead to long term capital
inflow and capital account improvement. As such there is an overall improvement in
BOP.
L: Thus, it is important for a country to attain low inflation.
4.

Full employment is a situation where all factors of production are fully utilised.
Unemployment of labour occurs when people who are willing and able to work are
unable to find jobs. Low unemployment allows the country to maximise output
produced and achieve efficiency in resource allocation. This is especially important for
resource-scarce Singapore, where the only resource is said to be labour.
Benefits of low unemployment rate
 P: Attaining low unemployment is important because it can promote economic
growth and raise standard of living.
E: If the economy can reduce unemployment to reach full employment, the previously
unemployed can produce something of a higher value for society. This increases
production as the economy moves from a point inside the PPC to a point on the PPC,
representing actual economic growth. There is also a strong link between employment
and consumer spending. As consumers confidence rise due to falling unemployment
rate, there is increased willingness of people to spend. With a rise in consumption,
aggregate demand rises, ceteris paribus. As a result, there is a multiple rise in national
income and a concomitant rise in standard of living.
L: Hence, it is important for a country to attain full employment.

5.

The balance of payments is a record of a countrys international transactions


between its residents and those of the rest of the world over a period of time. One of
the aims of the government is to maintain a healthy BOP, which is usually associated
with a balance of payments surplus, i.e. where the total value of credit transactions
exceeds the total value of debit transactions.

SRJC

38
Benefits of maintaining balance of payments equilibrium in the long term
 P: A healthy BOP means that the country is accumulating foreign exchange
reserves.
E: In times of need, these reserves can be used to finance purchase of Singapore
imports as well as maintaining the strength of the Singapore dollar (SGD) through
buying SGD and selling foreign currencies in the foreign exchange market. The
strength of the Singapore dollar has in turn safeguarded the nations domestic
purchasing power and helps to moderate increases in the cost of living. This thus
raises the standard of living of the people.
L: Hence, it is important for a country to attain a satisfactory BOP.
III. Conclusion
Achieving the macroeconomic goals has always been important. For Singapore, due to its
small, highly open economy with limited resources and high dependency on international
trade, the most important macroeconomic objective would be low inflation. With low inflation,
it increases the export competitiveness and attracts the much needed foreign direct
investment to Singapore. If this goal is missed, it is likely that the other macroeconomic goals
will not be easily attainable.
Suggested Marking Scheme
Level
Descriptors
L3
Clear criteria stated to determine the importance of achieving the 4 key
macroeconomic goals.
Detailed explanation of the key macroeconomic goals required, with good use of
examples, e.g. application to the Spores context.
Considered judgement on attainment of main economic goal for a country such as
Spore
L2
Thorough explanation of at least two of the four key macroeconomic goals required,
with some use of examples, e.g. good application to the Spores context, and key
criteria set
OR
Explanation of the key macroeconomic goals but with limited use of examples.
L1
Smattering answers with scant elaboration of its benefits

(b)
I.
1.

2.

Discuss whether developing countries have more to gain from globalisation than
developed countries.
[15]
Introduction
Definition: Globalisation refers to the process of continuing integration of the countries
in the world where national markets become increasingly interlinked. Globalisation
leads to greater interdependence between countries as there is greater mobility of
goods and services, capital, labour and technological know-how amongst countries. As
such, globalisation has exposed national economies to much more intense competition
than ever before.
Direction of essay: Developed countries are usually characterised by abundant supply
of skilled labour, capital goods and advanced technology, with comparative advantage
in producing capital-intensive goods. On the other hand, developing countries are
usually characterised by abundance supply of land and labour, with comparative
advantage in producing labour-intensive goods. Globalisation has the potential to allow
countries to gain, in terms of achieving the microeconomic and macroeconomic goals.
However, not all countries benefit equally.

SRJC

39
II.
Body
Thesis: Developing countries have more to gain than developed countries
3.
P: Reduced trade barriers help developing countries achieve economic growth
and favourable BOP.
E/E: With globalisation, there is increased trade as trade barriers are reduced.
Developing countries such as China open their market to international trade and
capital flow, making a breakthrough in producing low-end manufactured goods at lower
opportunity cost with her relative abundance of low-wage labour. As such, China is
able to gain comparative advantage (CA) in producing low-end manufactured goods as
compared to developed countries. With increase in export price competitiveness, and
assuming that demand is price elastic, it will lead to a more than proportionate
increase in quantity demanded for its exports. Hence total revenue increases. Coupled
with increased market access for export, this will lead to a rise in net exports. Hence
AE would increase, ceteris paribus. The resultant rise in national income would lead to
a rise in induced consumption, which would bring about a further increase in national
income. Hence, assuming economy is at less than full employment, there will be an
increase in production, employment and multiplied increase in national income. This is
especially beneficial for developing countries such as China as there would be greater
utilisation of unemployed or underemployed resources. In addition, the continuing
reallocation of manufacturing activities from industrialised to developing countries
opens new opportunity to expand trade. Furthermore, with rising net exports, ceteris
paribus, it will also lead to a favourable balance of payments for the developing
countries.
L: Thus, globalisation brings about economic growth and favourable BOP for
developing countries.
4.

P: On the other hand, there is a threat of unemployment in developed countries.


E/E: While developing countries gain jobs and foreign money as a result of
globalisation, the converse happens to developed countries. Manufacturing jobs, for
example, have been dramatically cut in the U.S. and Canada as a result of
globalisation. This means that while developing countries' citizens have more
opportunities, this comes at the expense of developed countries' labour markets.
Developed countries that have lost its CA to China would experience a fall in demand
for its low-end manufactured goods since Chinas low-end manufactured goods are
substitutes of their products. Hence developed countries low-end manufacturing
industry starts to decline due to lower profits obtained as a result of lower total
revenue. This fall in production activities leads to retrenchment of workers, especially
for low-skilled and older workers, thus resulting in structural unemployment in the
developed countries. This is especially so if the retrenched workers do not possess the
relevant skills to move into sunrise industries which are likely to be knowledge and
capital intensive. In addition, developed countries are conscious that the emerging
Asian economies are increasingly characterized by high skills and modern
technological know-how, a combination which will make them increasingly competitive
in high-tech markets and therefore in competition with highly skilled workers in
developed countries for performing high tech work, with a risk of job losses in the
developed economies. However, if the government is swift to implement appropriate
policies to equip the workforce with the necessary and relevant skills so as to stay
employable, structural unemployment issue can be mitigated, assuming the workers
are responsive to retraining and skills-upgrading programmes.
L: Hence, globalisation can bring about structural unemployment in developed
countries but the extent can be mitigated by relevant government policies.

SRJC

40
5.

P: There is also an increased inflow of foreign direct investment (FDI) from


developed countries to developing countries.
E/E: With globalisation, there is greater mobility of capital flow across countries. In
view of lower labour and land costs in developing countries, MNCs from developed
countries, especially those associated with labour-intensive operations, would invest in
these developing countries to tap on their comparative advantage. Developing
countries have much to gain from the inflow of FDI from the developed countries. With
increased FDI, it will lead to an increase in AD. In addition, the inflow of FDI into the
developing countries has resulted in exposure to new technology. With technological
transfer, productivity is raised. This lowers unit cost of production and raises AS. With
both an increase in AD and AS, developing countries can benefit from low-inflationary
sustained economic growth and falling unemployment. In addition, the inflow of longterm capital also improves capital account, ceteris paribus, leading to a favourable
balance of payments.
L: Hence, developing countries can benefit more than developed countries.

Anti-thesis: However, developed countries could have more to gain from globalisation
than developing countries.
6.
P: Globalisation can result in a worsening of the terms of trade (TOT) for
developing countries.
E/E: Developing countries may have a comparative advantage in primary products.
However, this may not offer a large scope for economic growth. The TOT for most
developing economies has been deteriorating. This means that year by year they have
to export more and more goods (especially primary agricultural products) to support a
given volume of imports. As such, their standard of living will not rise as fast. The
problem of weak TOT is due to the income inelastic nature of demand for their
products. Hence, even as globalisation brings about a rise in incomes, the demand for
primary agricultural products would only increase less than proportionately, ceteris
paribus. However, fuel-exporting developing economies have been in a much stronger
position, although in recent years, there has been considerable volatility in their TOT
as the world price of crude oil has fluctuated widely. On the other hand, for
developed countries, their TOT has experienced longer-term improvement. This
means that they have continued to receive increased real prices, especially for their
exports of manufactured consumer goods. Hence they are able to enjoy a greater
amount of import for a given export, leading to a higher standard of living. In addition,
their exports tend to be income-elastic. Hence with the developing countries enjoying
rising income from increased trade, this rise in income will lead to a more than
proportionate increase in developed countries exports, hence benefiting the
developed countries significantly, especially if such countries are dependent on
exports to drive their economic growth.
L: Thus, developing countries may not have more to gain from globalisation than
developed countries.
Anti-thesis: Additionally, both developing countries and developed countries may
suffer from a widening of income gap.
E/E: Globalisation has led to greater financial flows across national border. Total FDI
flows in the world increase. With the latter, it has resulted in exposure to new ideas,
technology and products. New technology, in both developing and developed
economies, creates greater demands for those with higher skills.
When there is an increase in demand for skilled labour in developing countries, it
raises the wages of these workers. This is made worse by the fact that with increased
labour mobility resulting from globalisation, it may give rise to a brain drain of skilled
workers in developing countries as highly qualified professionals migrate to developed
countries to benefit from the higher wages, thus worsening the severe skilled labour

SRJC

41
shortage. Hence with increase in wages of high-skills workers while low-skilled
workers income may remain constant or decrease, it will lead to a widening of income
inequality in the developing countries. For the developed countries, wages of workers
in high value-added sectors rise exponentially compared to wages of workers in low
skilled and low value-added sectors. Furthermore, with increased mobility of labour
from developing countries, it may also include the influx of foreign low-skilled workers
in search of better job prospects. In this way, the wages of low-skilled workers in the
developed countries may be further depressed. Hence with demand for skilled workers
outstripping its supply, wages continues to rise. This thus worsens its income gap.
L: Thus, developing countries may not have more to gain from globalisation than
developed countries as both countries would experience rise in inequity..
III. Conclusion
It is not necessarily true that developing countries have more to gain from globalisation than
developed countries. In fact, it has been argued that the benefits of globalisation can
outweigh its costs for most countries. However, these benefits and costs may not be
equally distributed amongst countries. The role of governments is thus instrumental. Good
foresight on sound and astute economic policies is needed to maintain the countrys
competitive edge in the global economy. Hence, whether a country can gain more from
globalisation as compared to another is dependent on the success of government policies to
maintain its international competitiveness, so as to maximize the opportunities presented by
globalisation and manage the challenges posed by globalisation.
Level
L4

L3

L2

L1

SRJC

Descriptors
For an accurate, balanced and well-developed explanation of the effects of
globalisation on developing and developed countries; in terms of who has
more to gain from globalisation
Use of relevant economic framework for thorough and clearly-explained
analysis
Able to appreciate the instrumental role of government in implementing
policies to reap the net benefits of globalisation so as to determine the extent
of gain enjoyed by the respective countries both developing and developed
countries.
Make a stand with insightful comments, with evidence of a substantiated
value-judgement made in terms of comparing whether developing or
developed economies have more to gain from globalisation
Accurate, balanced and clear explanation of the effects of globalisation on
developing and developed countries; in terms of who has more to gain from
globalisation
Use of relevant economic framework for clearly-explained analysis
Able to appreciate the extent of gain enjoyed by the respective countries
both developing and developed countries.
Make a stand with little justification
Answers tended to be skewed to one-side.
Some evidence of the use of economic framework to explain the effects of
globalisation
For an answer that shows some knowledge of the effects of globalisation in
general, with minimal reference made on whether developing countries have
more to gain from globalisation than developed countries.

42
Examiners comments:
OVERALL COMMENTS:
This is a manageable question and a total of 113 candidates have attempted this question. However, this quality
of answers provided by the majority of the candidates were not satisfactory as responses tended to lack detailed
analysis and clear end-points, suggesting a lack of preparation for this topic.

Economists forecast a world trade growth of 9.5% for 2010, with developing countries' trade growing
by 11% and developed countries' trade growing by 7.5%.
(a)

Explain why it is important for a country to attain its macroeconomic goals.

[10]

Strengths:
(1) Introduction
A large number of candidates were able to craft relatively good introductions, focusing on both (i)
identification of the 4 macroeconomic goals and (ii) reasons why the goals should be attained.
(2) Benefits of Economic Growth
A large number of candidates were able to explain the attainment of higher standard of living as the
benefit of achieving Economic Growth.
Areas for Improvement:
(1) Misinterpretation of Question
Part A requires candidates to justify the importance of attaining its macroeconomic goals by explaining
the benefits of the 4 macroeconomic goals. However, some candidates misinterpreted the question and
gave the following responses:
o Causes of macroeconomic problems
o Policies that can be implemented to attain the goals
(2) Lack of Scope in Response
A handful of candidates focussed solely on explaining the benefits of attaining 1 goal.
Some candidates thought that the end point of the question is standard of living, when the end-point
of the question was on benefits of growth. Hence their responses were limited, as they omitted all other
benefits of attainment of goals.
(3) Lack of Precision in Response
There was little attempt from candidates part to distinguish between the terms: material, and nonmaterial standard of living.
Instead of answering the question directly on the benefits of achieving the various macroeconomic
goal, students went the indirect way or explaining the cost of not achieving the goals.
(4) Conceptual Errors
A handful of candidates mistook Balance of Payments deficit to be Budget deficit.
o A balance of payments deficit occurs when the total value of debit transactions exceeds the
total value of credit transactions. This results in a net outflow of gold or foreign exchange
reserves and/or an increase in official borrowing
WHEREAS
o A budget deficit occurs when the government expenditure exceeds the revenue received.

What you wrote


High unemployment leads to a fall in productive
capacity and a lead to a fall in level of potential
growth.
A fall in level of unemployment will lead to a
movement from a point inside the PPC to a point
on the PPC.

SRJC

What it should be
High unemployment refers to a wastage of
resources and it refers to the economy producing
at a point within the PPC.
A fall in level of unemployment will lead to a
movement from a point inside the PPC to a point
closer to the PPC.

43
(b)

Discuss whether developing countries have more to gain from globalisation than developed
countries.
[15]

Strengths:
(1) Explanation of the Impact on Current Account
A large number of candidates were able to provide detailed explanation (including assumptions of PED
of X & M) of how globalisation impacted the current account.
(2) Use of diagrams (Keynesian and AD-AS Framework)
Though not many candidates used diagrams to support their explanations, those who used diagrams,
were able to provide detailed explanation, including the adjustment and multiplier process.
Areas for Improvement:
(1) Misinterpretation of Question and Conceptual Errors
A handful of students misinterpreted the requirements of the question because:
o A few candidates thought that Singapore is a developing country. Hence the entire essay
focused on comparing Singapore with US.
o Some candidates equated the characteristics of less developed countries to be openness of the
economy and the entire essay was focused on open versus closed economy.
(2) Lack of Scope in Responses
A large number of candidates were not able to provide sufficient scope as they focused on 1 or 2 points
(probably due to a lack of time). In addition, one of the more important points of structural
unemployment suffered by developed countries was left out by the majority of the candidates.
Many students did not provide clear a thesis and anti-thesis, hence there was no clear response to the
question and it resulted in a smattering of ideas and points
(3) Lack of precision in responses
A large number of responses lacked detailed economic analysis and were descriptive in nature,
somewhat resembling a GP or Geography essay. These candidates did not provide a clear end-point
and/or did not provide AD-AS analysis.
A number of candidates were not familiar with the response to the question as it required comparison
between developed and developing countries. Hence, these candidates merely explained costs and
benefits of globalisation in general and then merely ended the paragraphs with a stand on who gains
more without clear justification.
When comparing the extent of benefits between DCs and LDCs on Economic Growth, many students
did not link their analysis to the concept of Terms of Trade, hence, their analysis lacked rigour.

SRJC

44
6

I.
1.

2.

(a)

Explain how a change in a countrys labour productivity can influence its


balance of payments.
[8]

(b)

Assess whether a government should be concerned when faced with a deficit


in the budget balance and the current account balance.
[17]

Introduction
Define: Labour productivity is the ability to create goods and services from a given
amount of output. It is the efficiency of labour and is measured by output per unit of
labour input. The main drivers of labour productivity are skills, innovation and
investment. Balance of payments (BOP) is a record of a countrys international
transactions between its residents and those of the rest of the world over a period of
time. It records the flow of goods and services, gifts and assets between the
residents of a country and those of other countries over a period of time. One of the
aims of the government is to maintain a healthy balance of payments.
Direction of essay: Changes in a countrys labour productivity can influence its
balance of payments. A fall in labour productivity can adversely affect a countrys
balance of payments, and vice versa.

II.

Body

3.

P: A fall in labour productivity could lead to a worsening of an economys BOP.

4.

E/E: Assuming firms employ the same number of workers, if labour productivity falls,
this means that each worker is producing less. Hence there is a rise in the average
cost of production, resulting in a fall in aggregate supply (AS). This shifts the AS
curve to the left from ASo to AS1.
General price level

AS1
P1

AS0

P0

AD

5.

6.

0
Y0 Y1
Real national output
With the above fall in AS, this will lead to a shortage of goods and services at original
price P0 whereby the level of AS is less than the level of aggregate demand (AD).
There will be an upward pressure on prices within the economy. This will cause prices
to be bid up, and real national output will increase along the AS curve. At the same
time, there is a fall in the level of AD. This process of adjustment will continue until a
new equilibrium is reached at a higher general price level, from P0 to P1 and a fall in
the real national output from Y0 to Y1.
Hence a fall in labour productivity leads to a rise in its general price level. This will
increase prices of goods including price of exports. In a country such as Singapore,
due to its characteristics of being small, open and lacking natural resources, it is highly
dependent on exports to drive its economy. Hence with a loss in its export price

SRJC

45
competitiveness from a fall in labour productivity, being extremely export-oriented, this
will have an adverse effect on its current account. Since Singapores exports is price
elastic due to the availability of close substitutes produced by other countries, with a
rise in price of exports, it will result in a more than proportionate fall in quantity
demanded for exports. Hence there will be a fall in the total revenue of exports. On the
other hand, imports are relatively cheaper as compared to the higher-priced
domestically-produced substitutes as a result of the cost-push inflation in Singapore.
This will increase the demand for imports, hence leading to a rise in total expenditure
on imports. With a fall in net exports, ceteris paribus, it will lead to a worsening of its
current account.
7.

With a fall in labour productivity leading to a rise in unit labour costs, it means a higher
cost of production for firms. As such, it makes the country a less attractive place for
foreign direct investment (FDI) since expected profits fall. FDI is often seen to have
benefits, especially for a small, resource-scarce Singapore which is highly dependent
on FDI to sustain its economic growth. Hence with a fall in labour productivity, there
will be a fall in long-term investments into Singapore, leading to a worsening of its
capital account.

8.

L: With worsening current and capital account, a fall in labour productivity would lead
to a deterioration of a countrys BOP.

III. CONCLUSION
For economies like Singapore which only has a small pool of labour as the main resource,
labour productivity is an area that needs to be monitored closely at all times. This is because
a fall in labour productivity can have significant adverse impact on its BOP. Hence,
Singapore has focused a sizeable amount of resources to engage in supply-side policies in
order to raise labour productivity levels across the different sectors.
Suggested Marking Scheme:
Level
L3

Descriptors
Clear and thorough explanation of how falling productivity can lead to impact on
balance of payments, with a reasoned introduction and conclusion.
Reference to AD/AS diagram is clearly made and explained, with application to
an example such as Singapore.
Some explanation of how falling labour productivity can lead to impact on current
and capital account.
Some application to AD/AS framework
Some gaps in analysis
Smattering of points with vague understanding of how a fall in labour productivity
affects balance of payments outcomes. Conceptual error/s evident.

L2

L1

(b)
I.
1.
2.
3.

Assess whether a government should be concerned when faced with a deficit


in the budget balance and the current account balance.
[17]

Introduction
Define budget deficit: Excess of governments spending over its tax receipts
Define current account deficit: A deficit in the current account can occur when the total
expenditure on imports is more than the total revenue from exports.
Direction of essay: Whether a government should be concerned about the deficits is
dependent on its impact on the economic goals, the size of the deficit, the cause of the
deficit and the problems that can arise from deficit financing.

SRJC

46
II.

Body

Budget Deficit
Issue 1: Current economic conditions
4.
P: A budget deficit is a concern for the government when the economy has reached
a stage of near or at full employment, resulting in demand-pull inflation.
E/E: A budget deficit can arise from an increase in government expenditure and a fall
in tax rates. When personal income tax falls, there is a rise in disposable income and
hence a rise in consumption expenditure (C). When corporate income tax drops, there
is a rise in post-tax profits and hence a rise in investment expenditure (I). The rise in
G, C and I can lead to an increase in AD. As AD continues to increase and when the
economy reaches a stage of near or at full employment, domestic output cannot rise to
meet the surging demand. What results will be demand-pull inflation. When the
inflation gets excessive, it will reduce the export competitiveness of the country. This
can have an adverse effect on the balance of trade, given the price elastic demand for
the countrys exports. This can have an adverse impact for a country that is exportoriented and is heavily dependent on its export to drive its economy.
L: Thus, a deficit in the budget balance is a cause of concern for the government.
5.

P: However, in times of a recession, a budget deficit is not a cause of concern for


the government.
E/E: This is because if recession, characterised by falling GDP and rising
unemployment, persists, the falling incomes and loss of jobs can mean hardship
especially for those in the lower-income group. Thus there is a need for discretionary
fiscal polices to tackle the issue of recession. Expansionary polices need to be in
place, based on Keynes theory of spending out of a recession, with an increase in
government spending to pump prime the economy and a reduction in taxes (both
income and corporate taxes) to increase consumption and investment. The increase
in aggregate expenditure would then work through the multiplier to increase
production, create jobs and national income, and hence economic growth.
L: In this case, a budget deficit is not a cause of concern for the government.

Issue 2: Cause of the budget deficit


6.
P: Depending on the cause of the deficit, a budget deficit could be a concern for the
government.
E/E: If the deficit is caused by excessive spending in areas that do not have a direct
impact on the peoples standard of living such as in financing wars, an opportunity cost
is incurred, i.e. the funds could have been used to spend on economic development
that can raise the welfare of the citizens. In addition, it will also be a concern if the
budget deficit is caused by excessive subsidies and welfare benefits, which can be
counter-productive as the former perpetuate inefficiency whilst the latter can reduce
the motivation for people to actively seek jobs. An example would be the subsidies by
countries like Indonesia, Malaysia and Thailand on oil prices.
L: In this case, the government should be concerned when faced with a budget deficit.
7.

P: However, a budget deficit is not a concern for the government if the deficit is
caused by spending on building of physical infrastructure and augmenting of human
capital.
E/E: This is especially so for newly industrialising countries like Vietnam and
Cambodia where the governments may have to spend on the provision of efficient
airports, harbour facilities and transport systems. In addition, funds also have to be
spent in education to improve the literacy rate and perhaps helping companies
improve on their level of technology. They may have to incur huge deficits in the initial
stages where the tax revenue may not be enough to cover the immense expenditure.
However, these expenditures are necessary to attract FDI into the country so as to

SRJC

47
drive economic growth. More developed countries may also incur deficits in their bid to
maintain or enhance their countries competitive edge in the face of globalisation and
competition from low-wage countries.
L: Thus, the government should not be concerned when faced with a budget deficit.
Issue 3: How the deficit is financed
8.
Holding all other things constant, if the budget deficit is financed by available financial
resources from governments reserves, then it is not a concern for the government.
9.

P: However, it will be a concern for the government if the budget deficit is large and
financed by borrowing from private capital market.
E/E: This increase in demand for funds in the market will lead to a rise in interest rates.
The competition for funds will cause interest rates to rise and thus crowd out private
sector spending. This will in turn lower the level of private investment because of the
higher costs of borrowing funds. Hence there is a crowding-out effect. The crowding
out of private investment dampens the expansionary effect of government expenditure
on the economy, and thus be counter-productive.
L: Hence, in this case, a budget deficit is a cause of concern for the government.

Current Account Deficit


Issue 1: Negative impact on economic goals
13. P: A current account deficit is a concern for the government as it can cause a fall in
economic growth and employment.
E/E: A fall in export earnings and rise in import expenditure reduces the level of AE.
The multiplier process is activated when there is an initial fall in injection (i.e. exports).
An initial fall in injection will lead to an equal decrease in national income.
Consequently, this fall in incomes will lead to a fall in induced consumption and a fall in
withdrawals. The extent of fall in induced consumption is determined by the value of
marginal propensity to consume (MPC). The higher the value of MPC, the greater is
the fall in induced consumption and hence the larger is the decrease in national
income. The multiplier process continues until the initial fall in injection is equal to the
total decrease in withdrawals. Hence, the economy reaches a new equilibrium national
income at a lower level, hence leading to a fall in production, employment, national
income and hence actual economic growth.
14.

P: Additionally, a current account deficit can lead to a fall in the external value of
the countrys currency, leading to cost-push inflation
E/E: If the current account deficit is persistent, ceteris paribus, it means that total
currency outflow is greater than total currency inflow. As such, there is a surplus of
domestic currency in the foreign exchange market, hence leading to a fall in the
external value of the countrys currency. A depreciation of the countrys currency will
increase the price of imported inputs and the general price level. This problem of costpush inflation is more severe if the country is heavily dependent on imports of foodstuff
and raw materials such as Singapore. Since the import content of manufactured goods
stands at 63%, an increase in price of imported raw materials will result in Singapore
losing her export price competitiveness, hence worsening the current account deficit
problem. In addition, for countries with external debt, depreciation of currency will
increase the burden of servicing the external debt.
L: Since a current account deficit can lead to a fall in economic growth and a rise in
cost-push inflation, it is a cause of concern for the government.

Issue 2: Cause of the deficit


15. P: If the current account deficit is due to the loss of competitiveness of its goods and
services, then this is certainly a cause of concern for the government.

SRJC

48
E/E: This is because structural changes of the economy may be needed to find new
niche areas. This may lead to the problem of structural unemployment if the workers
are not flexible and receptive to the re-training to equip themselves with the relevant
skills.
16.

P: However, if the current account deficit is due to the heavy imports of capital and
investment goods, then this may not be a concern for the government.
E/E: This is because such imports of capital and investment goods are necessary for
the countrys industrialisation programme. When the country has achieved success in
its industrialisation programme, it may be able to produce manufactured goods for
export. Thus, in this case, a deficit may not be a problem for the country because as
the economy grows, it will be able to earn foreign exchange from its exports to pay for
its imports.
L: Thus, depending on what the cause is, the government may not be concerned
about the current account deficit.

Issue 3: How the deficit is financed


17. P: A current account deficit is a cause of concern because it can lead to the depletion
of foreign exchange reserves.
E/E: When a country has a persistent and large current account deficit, ceteris
paribus, it will lead to a sizable BOP deficit. This is a concern for the government as
it will have to run down its reserves of foreign exchange to finance the deficit. This
cannot go on indefinitely as the stock of official reserve assets is limited. The country
will find it difficult to make payments for imports if these reserves are depleted. If
essential imports become unavailable, the countrys economic growth and
development may be impeded. The impact will be especially significant for a country
which lacks natural resources and is highly dependent on imports. Singapores foreign
exchange reserves finance its imports and safeguards the value of currency. This is
crucial as the strength of the Singapore dollars has safeguarded the nations domestic
purchasing power which thus helps to moderate increase in cost of living. As such, it
will be a concern for the government if the foreign exchange reserves are continually
depleted to finance persistent current account deficit.
L: Hence, the government should be concerned when faced with a large and persistent
current account deficit.
III. Conclusion
A deficit in the budget balance and the current account balance is not a concern for the
government depending on the economic conditions faced by the country and the cause of
the deficits. Otherwise, both deficits are certainly a cause for concern, especially with the
problems that can arise from financing the deficit. In addition, if a government faces a
persistent twin deficit, it may compound the inflationary pressures faced by the countries
and also generate fears about the future stability of economy, thereby discouraging
potential investment and hence preventing the country from achieving its economic goals.
In such a situation, the government has to adopt relevant policies to address the deficits.
Suggested Marking Scheme
Level
L4

SRJC

Descriptors
Competent discussion that makes a weighed examination of each issue
discussed and good evaluative comments on whether the government is
concerned about the deficits, with clear link to end-point of question
Good application with relevant examples applying to different countries
Good use of AD/AS or Y=AE framework for economic analysis.
Provide an insightful conclusion

49
L3

L2

L1

SRJC

Good discussion of the criteria to determine whether the deficits are a concern
for the government, with evidence of a balanced view.
An answer that makes an adequate attempt to consider both sides of the
argument but answer tends to be more well-argued on only one side; or answer
lacks evaluative discussion of the arguments and counter-arguments to each
ISSUE presented above
Some application with relevant examples
Some valid response to the question but answer lacks consistency and rigour as
well as a comprehensive understanding of a deficit, its objectives and how is
financed
Analysis is skewed towards either concern or not a concern for the government
when faced with the deficit
Limited application with relevant examples
Some conceptual errors are evident
Descriptive answer that lacks economic analysis
Smattering if ideas, listing of consequences of budget deficit and current account
deficit, with limited link to question requirement

50
Examiners comments:
6
(a)
Explain how a change
balance of payments.

in

countrys

labour

productivity

can

influence

its
[8]

I.
1.

Question Interpretation
Generally students understood the question requirement, with many achieving Level 2 marks. To reach a
higher score, students could demonstrate application skills via useful and relevant examples, with a
reasoned conclusion.

2.

End point of question is not clearly addressed


NOT about macroeconomic goals
Not current or/and capital account per se

3.

Incomplete coverage, with many students explaining the impact on current account, and ignoring the
impact on capital account

Clarity and Precision/Content Knowledge And Application


What you wrote
What it should be
Balance of payments (BOP) is a record of a countrys
4.
Definition of balance of payments
Refers to the net inflow and outflow of money international transactions between its residents and
those of the rest of the world over a period of time.
with the foreign sector
Is a record of total receipts of trade between its It records the flow of goods and services, gifts and
assets between the residents of a country and those of
citizens and the rest of the world
Refers to the difference between a countrys other countries over a period of time
credit transaction and debit transactions
II.

5.

Effect of PED on total revenue


Assume price elasticity of demand for export is
price elastic, a fall in price of export will lead
to a more than proportionate increase in the
total revenue

Assume price elasticity of demand for export is more


than one, a fall in price of export will lead to a more
than proportionate increase in its quantity
demanded, hence increasing its total revenue

Inappropriate use of economic terms


Balance of payments increases / rises
Price of exports is cheaper
Increase in imports and fall in exports
7.
Lack of clarity and assumptions and limited
analysis
Ceteris paribus, when export earnings fall, the
balance of payments will deteriorate

Improvement in the balance of payments account


Exports are cheaper
Increase in imports expenditure and fall in
exports revenue
The gaps must be addressed to raise the quality of
analysis:
(i) State explicitly what ceteris paribus means in this
context.
(ii) It should be firstly: import expenditure on goods
and services unchanged, current account
deteriorates
(iii) Assuming capital account unchanged, the balance
of payments which consists of both the current
and capital account will worsen.

6.

In addition, you should also attempt to analysis the


impact of a change in price of domestic goods on
demand for import substitutes and hence import
expenditure
(b)

I.
1.

Assess whether a government should be concerned when faced with a deficit in the budget
balance and the current account balance.
[17]

Question Interpretation
The quality of the answers varied. Weak answers were not able to go beyond Level 2 marks because
they were not able to fully comprehend question requirement.

SRJC

51
2.

Trigger is on current account deficit and NOT on current account balance. Hence trigger should NOT
include current account surplus.

3.

End point is not clearly addressed


NOT about the policies to solve the respective deficits

4.

Incomplete analysis
One-sided analysis, explaining only the thesis and ignoring the anti-thesis (ie why the government should
NOT be concerned)
Not just about the impact on macroeconomic goals
Not just about the causes of the current account deficits
Weak answer demonstrated little evidence of the use of AD/AD or Y-AE framework for economic
analysis.
Little application to different countries via relevant examples

Clarity and Precision/Content Knowledge And Application


What you wrote
What it should be
5.
Definition of Budget Deficit
Budget deficit will lead to debts incurred only if the
Refers to government incurring a debt
government does not have sufficient reserves to
fund the deficit.
Refers to a situation when government has
insufficient funds to repay external debt
Hence budget deficit simply refers to excess of
governments spending over its tax receipts
When government borrows from the private capital
6.
Crowding Out effect
When government borrows, this increases market, this increase in demand for funds in the
supply of money, hence interest rate increases.
market will lead to a rise in interest rates. The
(Qn: where is the crowding out effect?)
competition for funds will cause interest rates to rise
and thus crowd out private sector spending. This will
in turn lower the level of private investment because
of the higher costs of borrowing funds. Hence there is
a crowding-out effect. It pushes away/crowds out
private investment.

II.

SRJC

You might also like